INFECTO Flashcards

1
Q

EN LA JURISDICCIÓN SANITARIA DONDE USTED LABORA, SE LE ENCOMIENDA REALIZAR UNA PLÁTICA SOBRE ZOONOSIS. LA PLÁTICA DEBERÁ COMENZAR CON LA ZOONOSIS QUE TENGA LA MÁS ALTA TASA DE MORTALIDAD., QUE ES?

A

RABIA
La RABIA tiene una distribución mundial. En 2007 fue reportado en 103 países. En los perros se reporta 54 % de la rabia animal. La epidemiología de rabia en humanos refleja aquella de la de animales locales.

En países en desarrollo en donde la rabia canina es común la mayoría de los casos humanos resulta de la mordedura de perros con la enfermedad. Y en aquellos en donde los perros están inmunizados resulta de animales silvestres con rabia.

La Organización Mundial de la Salud, estima que al año fallecen 55 000 humanos a causa de rabia y la incidencia anual de la enfermedad es de 100 000 casos.

LA RABIA ES LA ZOONOSIS DE DISTRIBUCIÓN MUNDIAL CON MAYOR TAZA DE INCIDENCIA. ADEMÁS DEBES CONSIDERAR QUE LA RABIA ES UNA ZOONOSIS DE DISTRIBUCIÓN MUNDIAL CONSTANTE. (NOM-011-SSA2-2011, PARA LA PREVENCION Y CONTROL DE LA RABIA HUMANA Y EN LOS PERROS Y GATOS).

How well did you know this?
1
Not at all
2
3
4
5
Perfectly
2
Q

se define por la presencia de bacterias viables en sangre. Se puede aplicar igualmente a la presencia de otros microorganismos (viremia, fungemia o Parasitemia).

A

Bacteriemia

How well did you know this?
1
Not at all
2
3
4
5
Perfectly
3
Q

Síndrome de respuesta inflamatoria sistémica (SRIS): respuesta inflamatoria desencadenada por diversos procesos. Se caracteriza por la presencia de 2 o más de los siguientes datos:

A

T> 38 o <36, FC> 90, FR > 20 y leucocitos > 12 mil o < 4 mil con 10% de formas inmaduras.

How well did you know this?
1
Not at all
2
3
4
5
Perfectly
4
Q

disfunción orgánica que pone en riesgo la vida desencadenada por una respuesta mal regulada a
una infección

A

Sepsis

How well did you know this?
1
Not at all
2
3
4
5
Perfectly
5
Q

sepsis en la que a pesar de un correcto aporte de fluidos persiste la hipotensión y la hipoperfusión periférica, requiriendo maneo con inotrópicos y/o vasopresores, además de presencia de
disfunción orgánica.

A

Choque séptico

How well did you know this?
1
Not at all
2
3
4
5
Perfectly
6
Q

llamado así porque no suele asociar bacteriemia, sino un efecto de una toxina. El cuadro está mediado por la acción de superantígenos que desencadenan la liberación descontrolada de citocinas proinflamatorias

A

Choque tóxico

El prototipo es el choque tóxico estafilocócico mediado por la TSST-1 de S. aureus.

How well did you know this?
1
Not at all
2
3
4
5
Perfectly
7
Q

El diagnóstico de bacteriemia se realiza mediante

A

hemocultivo

How well did you know this?
1
Not at all
2
3
4
5
Perfectly
8
Q

Los microorganismos más frecuentemente asociados a catéteres y dispositivos endovasculares son

A

estafilococos coagulasa negativos (S. epidermidis), Candida, S. aureus y algunos gramnegativos.

How well did you know this?
1
Not at all
2
3
4
5
Perfectly
9
Q

etiología de la ENDOCARDITIS INFECCIOSA

A

S. aureus es el más frecuente de endocarditis.

How well did you know this?
1
Not at all
2
3
4
5
Perfectly
10
Q

etiología de la ENDOCARDITIS INFECCIOSA por protésica precoz (primer año post cirugía).

A

S. epidermidis

How well did you know this?
1
Not at all
2
3
4
5
Perfectly
11
Q

En usuarios de drogas parenterales (UDVP) el germen más frecuente que causa endocarditis es

A

S. aureus (+fc), Pseudomonas y Serratia

How well did you know this?
1
Not at all
2
3
4
5
Perfectly
12
Q

en endocarditis subaguda la etiología mas frecuente es

A

-S. viridans es el más común de

How well did you know this?
1
Not at all
2
3
4
5
Perfectly
13
Q

son causa de endocarditis infecciosa con hemocultivo negativo, ya que precisan medios de cultivo enriquecidos e incubación prolongada.

A

microorganismos del grupo HACEK

How well did you know this?
1
Not at all
2
3
4
5
Perfectly
14
Q

son causantes de endocarditis con hemocultivos negativos sobre todo en válvula aórtica

A

Coxiella y Brucella

How well did you know this?
1
Not at all
2
3
4
5
Perfectly
15
Q

constituye una causa poco frecuente de embolismo sistémico que suele asociarse a enfermedades malignas y estados de hipercoagulabilidad (SAF o LES). Consiste en la presencia de un trombo estéril adherido al endocardio valvular.

A

La endocarditis trombótica no bacteriana (Libman-Sacks)

How well did you know this?
1
Not at all
2
3
4
5
Perfectly
16
Q

debe sospecharse la presencia de endocarditis infecciosa en las siguientes situaciones

A
  • Paciente con fiebre sin focalidad infecciosa aparente.
  • Paciente febril con soplo cardíaco.
  • Paciente con fiebre y fenómenos embólicos (infarto, hemorragia) o lesiones cutáneas (estigmas periféricos cutáneos de endocarditis).
How well did you know this?
1
Not at all
2
3
4
5
Perfectly
17
Q

Las lesiones de Janeway (maculopápulas eritematosas en palmas y plantas) son manifestaciones clínicas de

A

embolismos periféricos.

How well did you know this?
1
Not at all
2
3
4
5
Perfectly
18
Q

a localización más frecuente de endocarditis infecciosa es la válvula

A

mitral, seguida de la aórtica y en tercer lugar ambas al mismo tiempo.

How well did you know this?
1
Not at all
2
3
4
5
Perfectly
19
Q

En usuarios de drogas parenterales es más frecuente la endocarditis en válvula

A

tricúspide. Suele ser un cuadro hemodinámico menos grave y los émbolos generalmente son pulmonares.

How well did you know this?
1
Not at all
2
3
4
5
Perfectly
20
Q

Criterios empleados para estatificar la posibilidad

diagnóstica de endocarditis

A

modificados de Duke, propuestos por Durack

Criterios clínicos: 2mayores, 1mayor y 3menores o 5menores.

How well did you know this?
1
Not at all
2
3
4
5
Perfectly
21
Q

criterios MAYORES modificados de Duke para ENDOCARDITIS

A
  1. Hemocultivos positivos: a) Microorganismos típicos en al menos 2 hemocultivos separados, b) Hemocultivos persistentemente positivos y c) Serología positiva para Coxiella (antígenos de fase I).
  2. Hallazgos en ecocardiografía: a) Vegetación o chorro valvular, absceso o dehiscencia en prótesis o b)
    Nueva regurgitación valvular.
  3. Serología positiva para Coxiella (antígenos de fase I)
How well did you know this?
1
Not at all
2
3
4
5
Perfectly
22
Q

criterios MENORES modificados de Duke para ENDOCARDITIS

A
  1. UDVP o cardiopatía predisponente.
  2. Fiebre > 38°C.
  3. Fenómenos vasculares: émbolos en arterias mayores, infartos sépticos pulmonares, aneurismas
    micóticos, manchas de Janeway o hemorragia intracraneal/conjuntival.
  4. Fenómenos inmunológicos: glomerulonefritis, nódulos de Osler, manchas de Roth, factor reumatoide.
  5. Ecocardiografía sugestiva sin ser criterio mayor.
  6. Hallazgos microbiológicos sin ser criterios mayores.
How well did you know this?
1
Not at all
2
3
4
5
Perfectly
23
Q

Tx Endocarditis tricúspidea no complicada: suele limitarse a 2 semanas

A

Cloxacilina + aminoglucósido

How well did you know this?
1
Not at all
2
3
4
5
Perfectly
24
Q

tx de endocarditis por Estafilococo en válvula nativa:

A

Cloxacilina + aminoglucósido, Si es MRSA se recomienda vancomicina.

How well did you know this?
1
Not at all
2
3
4
5
Perfectly
25
Q

tx de endocarditis por Estafilococo en prótesis

A

cloxacilina 6 semanas + aminoglucósido 2.

Si es MRSA vancomicina 6 semanas, rifampicina 6 semanas y aminoglucósido 2 semanas

How well did you know this?
1
Not at all
2
3
4
5
Perfectly
26
Q

tx de endocarditis por Estreptococos sensibles a penicilina

A

penicilina G durante 4 semanas + aminoglucósido.

How well did you know this?
1
Not at all
2
3
4
5
Perfectly
27
Q

tx de endocarditis por germenes HACEK

A

cefalosporina de 3era generación. En alergias quinolona.

How well did you know this?
1
Not at all
2
3
4
5
Perfectly
28
Q

tx de endocarditis por Bacilos gramnegativos

A

ampicilina o cefalosporina de 3era generación + aminoglucósido.

How well did you know this?
1
Not at all
2
3
4
5
Perfectly
29
Q

tx de endocarditis por Brucella

A

doxiciclina + rifampicina + cotrimoxazol.

How well did you know this?
1
Not at all
2
3
4
5
Perfectly
30
Q

tx de endocarditis por Coxiella

A

Doxiciclina + hidroxicloroquina.

How well did you know this?
1
Not at all
2
3
4
5
Perfectly
31
Q

tx de endocarditis por Hongos

A

anfotericina B + azólicos.

How well did you know this?
1
Not at all
2
3
4
5
Perfectly
32
Q

es el único germen asociado a la fiebre

reumática

A

Streptococcus B-hemolítico del grupo A

por lo que un tratamiento agresivo con Penicilina previene los brotes de fiebre reumática.

How well did you know this?
1
Not at all
2
3
4
5
Perfectly
33
Q

EPIDEMIOLOGIA DE LA FIEBRE REUMATICA

A

Edad frecuente de Fiebre reumática es entre 5 a 15 años de edad, en pacientes sobretodo que viven en HACINAMIENTO Y TIENEN NIVEL SOCIOECONÓMICO BAJO

How well did you know this?
1
Not at all
2
3
4
5
Perfectly
34
Q

CUADRO CLINICO TIPICO DE LA FIEBRE REUMÁTICA

A

2 a 3 semanas posteriores a la Faringoamigdalitis con comienzo de fiebre, mialgias, artralgias, hiporexia.
 Artritis en fiebre reumática: Dolor, enrojecimiento, en tobillos, codos y rodillas. Además de interfalángicas. Esto se debe a depósito de complejos inmunes en la cavidad articular.
 Corea de Sydenham: Danzante: movimientos involuntarios de extremidades, músculos de la cara
 Nódulos de Meynet: Nódulos subcutáneos no dolorosos, móviles en superficies extensoras.
 Carditis y otras alteraciones. : Miocarditis caracterizada por datos de miocardiopatía dilatada

How well did you know this?
1
Not at all
2
3
4
5
Perfectly
35
Q

¿Cómo ATACA LA FIEBRE REUMATICA AL CORAZON?

A

causan alteración en el sarcolema cardiaco

How well did you know this?
1
Not at all
2
3
4
5
Perfectly
36
Q

patología cardiaca de la fiebre reumática

A
  • Miocardiopatía dilatada
  • Pericarditis
  • bloqueo AV de primer grado.
  • Insuficiencia y después estenosis mitral
How well did you know this?
1
Not at all
2
3
4
5
Perfectly
37
Q

es la valvulopatía más frecuente en la fiebre reumática

A

Doble lesión mitral

How well did you know this?
1
Not at all
2
3
4
5
Perfectly
38
Q

Orden de lesión valvular en la fiebre reumatica

A
  1. Mitral 2. Aórtica 3. Tricuspídea 4. Aórtica
How well did you know this?
1
Not at all
2
3
4
5
Perfectly
39
Q

Nivel positivo para AELOS en fiebre reumatica

A

> 333 UI

How well did you know this?
1
Not at all
2
3
4
5
Perfectly
40
Q

¿Cuánto tiempo duran elevadas las AELOS en fiebre reumática ?

A

3 MESES

How well did you know this?
1
Not at all
2
3
4
5
Perfectly
41
Q

criterios para DIAGNOSTICO DE FIEBRE REUMÁTICA

A

jones
SE REALIZA CON LA PRESENCIA DE 2
CRITERIOS MAYORES, 1 CRITERIO MAYOR 2 MENORES o 3 MENORES

How well did you know this?
1
Not at all
2
3
4
5
Perfectly
42
Q

MANEJO INICIAL DE LA FIEBRE REUMATICA

Dolor articular

A

ASA como primera elección en la fiebre reumática aguda. Se puede agregar NAPROXENO que también ha sido utilizado para el dolor.

How well did you know this?
1
Not at all
2
3
4
5
Perfectly
43
Q

MANEJO INICIAL DE LA FIEBRE REUMATICA

Corea de Sydenham

A

Carbamazepina

How well did you know this?
1
Not at all
2
3
4
5
Perfectly
44
Q

MANEJO INICIAL DE LA FIEBRE REUMATICA

antibioticoterapia

A

Penicilina 500mg VO cada 12hrs

Penicilina Benzatinica 1,200,000 UI IM

How well did you know this?
1
Not at all
2
3
4
5
Perfectly
45
Q

MANEJO INICIAL DE LA FIEBRE REUMATICA

carditis

A

Prednisona o Prednisolona como primera elección

How well did you know this?
1
Not at all
2
3
4
5
Perfectly
46
Q

MANEJO profiláctico DE LA FIEBRE REUMATICA

A

Eliminar factores de riesgo como la higiene, hacinamiento.
ATB iniciada los primeros 10 días de la faringitis
Se puede dar Penicilina Benzatinica 600,000 UI en niños y a 1,200,000 en adultos

How well did you know this?
1
Not at all
2
3
4
5
Perfectly
47
Q

El tratamiento de faringoamigdalitis estreptocócicas es

A

penicilina G benzatínica en monodosis por vía IM o bien penicilina V por vía oral durante 10 días.
En caso de alergia a la penicilina se pueden utilizar macrólidos, cefalosporinas o clindamicina

How well did you know this?
1
Not at all
2
3
4
5
Perfectly
48
Q

transmision de la difteria

A

via area

How well did you know this?
1
Not at all
2
3
4
5
Perfectly
49
Q

complicaciones de la difteria

A

a. Miocarditis diftérica: las manifestaciones clínicas de miocarditis son infrecuentes, apareciendo en la 2-3 semana en forma de insuficiencia cardíaca o arritmias.
b. Sistema nervioso: afecta fundamentalmente a NC o nervios periféricos entre las 2-6 semanas. La forma más frecuente es parálisis del paladar blando, seguido de oculomotor.

How well did you know this?
1
Not at all
2
3
4
5
Perfectly
50
Q

diagnóstico de difteria

A

cultivo faríngeo en medio selectivo de Tinsdale donde crece formando colonias negras

How well did you know this?
1
Not at all
2
3
4
5
Perfectly
51
Q

tratamiento de difteria

A

macrólidos (eritromicina de preferencia).

How well did you know this?
1
Not at all
2
3
4
5
Perfectly
52
Q

cuadro clínico de la difteria

A

depende de la localización anatómica de las lesiones y de la gravedad del proceso tóxico.

  • Difteria faríngea: presenta placas blanquecinas y adenopatías cervicales.
  • Difteria laríngea: se manifiesta por tos, disnea, ronquera y obstrucción de vía aérea.
  • Difteria nasal: secreción sanguinolenta con costras.
  • Difteria cutánea: úlceras crónicas no cicatrizales.
How well did you know this?
1
Not at all
2
3
4
5
Perfectly
53
Q

Epiglotitis: se produce sobre todo por

A

Haemophilus influenzae tipo B y neumococo. Puede tener curso progresivo y potencialmente fatal por obstrucción de la vía aérea.

How well did you know this?
1
Not at all
2
3
4
5
Perfectly
54
Q

tratamiento de la epiglotitis

A

Se trata con cefalosporinas, quinolonas y esteroides.

How well did you know this?
1
Not at all
2
3
4
5
Perfectly
55
Q

criterios de Murray son para

A

métodos diagnósticos no invasivos para dx de neumonia
Examen de esputo: la tinción de Gram y el cultivo de la muestra sigue siendo útil siempre que presente >25 PMN y < 10 células epiteliales por campo

How well did you know this?
1
Not at all
2
3
4
5
Perfectly
56
Q

factor de virulencia de TUBERCULOSIS

A

cordfactor

How well did you know this?
1
Not at all
2
3
4
5
Perfectly
57
Q

que es el complejo primario de Gohn

A

expresión radiológica de la infección por M. tuberculosis en los macrófagos alveolares que no son capaces de contener y eliminar al bacilo, produciendo una replicación a nivel alveolar con posterior diseminación mediante los vasos linfáticos hasta ganglios regionales. lo que origina el denominado “complejo primario de Gohn” (neumonitis más linfangitis más adenitis)

How well did you know this?
1
Not at all
2
3
4
5
Perfectly
58
Q

constituye el principal factor de riesgo para el desarrollo de enfermedad tuberculosis

A

VIH
Generalmente la activación se da en los primeros 2 años de la primoinfección.
Otros factores de riesgo son neoplasias, insuficiencia renal o hepática, desnutrición, DM, tabaquismo, terapia inmunosupresora o hacinamiento.

How well did you know this?
1
Not at all
2
3
4
5
Perfectly
59
Q

DIAGNOSTICO DE TB

A

Prueba de la tuberculina
IGRA
Técnicas microbiológicas directas:

How well did you know this?
1
Not at all
2
3
4
5
Perfectly
60
Q

en que consiste la prueba de tuberculina

A

la intradermorreacción de Mantoux está constituida por inmunidad celular específica que responde a la inyección intradérmica de un conjunto de proteínas denominado PPD (derivado proteico purificado) que contiene proteínas comunes a M. tuberculosis, al BCG y a algunas micobacterias.

How well did you know this?
1
Not at all
2
3
4
5
Perfectly
61
Q

cuando se considera positiva la prueba de tuberculina

A
  • con >5mm de induración a las 48-72 horas si px tuvo: contacto estrecho con enfermo activo, coinfección con VIH, uso de esteroides, historia de trasplantes, cambios en radiografía o hallazgos clínicos.
  • o con >10mm en sospecha de TB activa sin las características anteriores
How well did you know this?
1
Not at all
2
3
4
5
Perfectly
62
Q

la prueba de tuberculina indic a

A

la existencia de inmunidad cutánea frente a M. tuberculosis que a su vez puede haberse adquirido tras la infección previa, mediante vacunación o tras el contacto con determinadas micobacterias no patógenas.

How well did you know this?
1
Not at all
2
3
4
5
Perfectly
63
Q

en que consiste la IGRA: ensayos de liberación de interferón-Y (quantiferón) para dx de Tb

A

en la detección y cuantificación en suero del interferón-y sintetizado por los linfocitos T de memoria, activados tras ser expuestos a antígenos de M. tuberculosis. Generalmente, son más específicos que la prueba de la tuberculina

How well did you know this?
1
Not at all
2
3
4
5
Perfectly
64
Q

El diagnóstico definitivo de la enfermedad tuberculosa pasa por la demostración de M. tuberculosis en

A
alguna muestra biológica del paciente tras un cultivo en medios específicos: Löwenstein-Jensen (sólido) 
o Middlebrook (líquido).
How well did you know this?
1
Not at all
2
3
4
5
Perfectly
65
Q

La presencia de BAAR se realiza en tinciones de

A

Ziehl-Neelsen o auramina-rodamina es muy sugestiva de tuberculosis, particularmente en un contexto clínico y radiológico apropiado.

How well did you know this?
1
Not at all
2
3
4
5
Perfectly
66
Q

Osteomielitis tuberculosa: fundamentalmente afecta a

A

la columna dorsal (mal de Pott). Pueden existir

abscesos. La tuberculosis articular afecta fundamentalmente a grandes articulaciones de carga.

How well did you know this?
1
Not at all
2
3
4
5
Perfectly
67
Q

constituye la forma más frecuente de tuberculosis

extrapulmonar

A

Adenitis tuberculosa
Puede aparecer como enfermedad localizada fundamentalmente en el cuello (escrófula) o en forma de adenopatías generalizadas. Este ganglio suele tener consistencia gomosa y fistuliza a piel drenado de forma espontánea material caseoso.

How well did you know this?
1
Not at all
2
3
4
5
Perfectly
68
Q

Tratamiento de la enfermedad tuberculosa:

A
  • primera fase de tratamiento (primeros dos meses) para asegurar eficacia -> isoniazida, rifampicina, pirazinamida y etambutol.
  • segunda fase, con al menos dos drogas, hasta completar los seis meses -> isoniacida + rifampicina
How well did you know this?
1
Not at all
2
3
4
5
Perfectly
69
Q

es el fármaco más importante para tratar TB, es un Bactericida al inhibir la síntesis de ácidos micólicos.

A

Isoniacida

How well did you know this?
1
Not at all
2
3
4
5
Perfectly
70
Q

efecto adverso de la Isoniacida

A

 Presenta metabolismo hepático.
-La hepatotoxicidad es su efecto adverso más importante; hasta en 10% hay elevación de transaminasas, si se multiplican 5 veces sus valores normales requiere de suspensión.
 Neuropatía periférica (disminución de vitamina B6)
 Hipersensibilidad
 Hiperuricemia
 Agranulocitosis

How well did you know this?
1
Not at all
2
3
4
5
Perfectly
71
Q

es un bactericida en tto para Tb al inhibir la síntesis de ARN. Actúa sobre las poblaciones de bacilos en
replicación activa.

A

Rifampicina

El principal efecto adverso es la HEPATOTOXICIDAD.

How well did you know this?
1
Not at all
2
3
4
5
Perfectly
72
Q

efectos adversos de pirazinamida

A

hiperuricemia, hepatotoxicidad y fiebre.

How well did you know this?
1
Not at all
2
3
4
5
Perfectly
73
Q

es un bacteriostático que inhibe la síntesis de la pared celular y ARN bacteriano, se da en tto para Tb

A

Etambutol

How well did you know this?
1
Not at all
2
3
4
5
Perfectly
74
Q

efectos adversos de Etambutol

A

neuritis óptica, neuropatía periférica e hiperuricemia.

How well did you know this?
1
Not at all
2
3
4
5
Perfectly
75
Q

es un aminoglucósido con efecto bactericida. Se reserva para casos de resistencia a Tb

A

estreptomicina

How well did you know this?
1
Not at all
2
3
4
5
Perfectly
76
Q

cuantas dosis son en el tratamietno primaria para tb

A
105
FASE INTENSIVA (2 meses) (60 dosis)
FASE DE SOSTÉN (4 meses) (45 dosis)
How well did you know this?
1
Not at all
2
3
4
5
Perfectly
77
Q

QUE SE HACE SI ES POSITIVA ALGUNA BACILOSCOPIA PARA TB

A

SE INICIA TRATAMIENTO ANTITUBERCULOSO POR 6 MESES (25 SEMANAS)

How well did you know this?
1
Not at all
2
3
4
5
Perfectly
78
Q

Cuando el paciente que tenía el tratamiento primario lo abandona o recae al presentar alguna baciloscopia positiva, deberá

A

recibir re-tratamiento primario

(150)

How well did you know this?
1
Not at all
2
3
4
5
Perfectly
79
Q

FARMACOS DE RETRATAMIENTO PRIMARIO (150 dosis)

A

FASE INTENSIVA (60 dosis)
Isoniacida + Rifampicina + Pirazinamida + Etambutol + Estreptomicina
FASE INTERMEDIA(30 dosis)
Isoniacida + Rifampicina + Pirazinamida + Etambutol + Estreptomicina
FASE DE SOSTÉN (30 dosis)
Isoniacida + Rifampicina + Pirazinamida + Etambutol +
Estreptomicina

How well did you know this?
1
Not at all
2
3
4
5
Perfectly
80
Q

Cuando el paciente que recibió el retratamiento primario fracasa o es abandonado, se deberá

A

evaluar como una posible Tuberculosis farmacorresistente en los estados donde se definirá el manejo.

How well did you know this?
1
Not at all
2
3
4
5
Perfectly
81
Q

Tuberculosis multidrogoresistente (MDR) se refiere a

A

Cuando hay resistencia a H y R (ambos).
La tuberculosis extremadamente resistente se define como TB MDR + resistencia a una fluoroquinolona y al menos 1 de las 3 drogas inyectables.

How well did you know this?
1
Not at all
2
3
4
5
Perfectly
82
Q

TB EXTRAPULMONAR QUE ES CAUSA DE NOTIFICACIÓN INMEDIATA <24HORAS.

A

TB meníngea

Se asienta en la base encefálica formando microcranulomas con ruptura. Puede ser un curso
crónico o subagudo con rigidez de nunca, signos meníngeos, letargia y déficit neurológico. LCR
muestra linfocitosis, hipoglucorraquia e hiperproteinorraquia. Se deben utilizar esteroides en su tratamiento, Se debe realizar cultivo de LCR o baciloscopia para el diagnóstico.

How well did you know this?
1
Not at all
2
3
4
5
Perfectly
83
Q

A TODO PACIENTE CON VIH SE DEBE REALIZAR BACILOSCOPIAS Y PPD. En caso de PPD positiva
pero baciloscopia negativa: Quimioprofiaxis con

A

Isoniacida

How well did you know this?
1
Not at all
2
3
4
5
Perfectly
84
Q

son la causa más frecuente de diarrea en niños, y cual en adultos

A

Los rotavirus son la causa más frecuente en niños y el virus Norwalk en adultos.

How well did you know this?
1
Not at all
2
3
4
5
Perfectly
85
Q

Bacterias productoras de neurotoxinas: se denominan neurotoxina porque

A

predomina su acción a nivel del hipotálamo, sobre el área del vomito, de modo que su clínica incluye náuseas y vómitos.
Se suele adquirir la toxina preformada con los alimentos por lo que la clínica es precoz tras la intesta. Los agentes que mayormente se asocian son Bacillus cereus y S. aureus

How well did you know this?
1
Not at all
2
3
4
5
Perfectly
86
Q

son Bacterias productoras de neurotoxinas

A

Bacillus cereus y S. aureus

How well did you know this?
1
Not at all
2
3
4
5
Perfectly
87
Q

Bacterias productoras de enterotoxinas: este tipo de toxinas actúan a nivel de

A

la superficie de los enterocitos sin destrucción de la mucosa; alteran el intercambio iónico y favorecen el paso de agua libre hacia la luz intestinal.
Provocan diarrea acuosa, sin productos patológicos.
La toxina se produce habitualmente in vivió.
Este tipo de diarrea suele ser por V. cholerae (cólera; agua de arroz), también B. cereus, C. perfingens y E. coli enterotóxica (etiología más frecuente de la diarrea del viajero).

How well did you know this?
1
Not at all
2
3
4
5
Perfectly
88
Q

son Bacterias productoras de enterotoxinas

A

V. cholerae (cólera; agua de arroz), también B. cereus, C. perfingens y E. coli enterotóxica (etiología más frecuente de la diarrea del viajero).

How well did you know this?
1
Not at all
2
3
4
5
Perfectly
89
Q

Bacterias productoras de citotoxinas: estas toxinas reciben su nombre porque

A

destruyen las células, lo que causa una mayor inflamación a nivel local y provocan diarrea de tipo disentería (moco, sangre y leucocitos en las heces).
Pueden cursar con fiebre.
Son ejemplos: Shigella, E. coli enterohemorrágico y
C. difficile.

How well did you know this?
1
Not at all
2
3
4
5
Perfectly
90
Q

son Bacterias productoras de citotoxinas

A

Shigella, E. coli enterohemorrágico y

C. difficile.

How well did you know this?
1
Not at all
2
3
4
5
Perfectly
91
Q

bacterias que suelen asociarse como complicación un síndrome urémico hemolítico.

A

Shigella y E. coli 0157:H7

How well did you know this?
1
Not at all
2
3
4
5
Perfectly
92
Q

es el agente etiológico más frecuente de la diarrea nosocomial.

A

Clostridium difficile

How well did you know this?
1
Not at all
2
3
4
5
Perfectly
93
Q

manifestación característica del Clostridium difficile

A

colitis pseudomembranosa

How well did you know this?
1
Not at all
2
3
4
5
Perfectly
94
Q

tratamioento del Clostridium difficile

A

metronidazol o vancomicina VO.

How well did you know this?
1
Not at all
2
3
4
5
Perfectly
95
Q

son Bacterias enteroinvasivas

A

Campylobacter, Shigella, Salmonella y E. coli.

directamente invaden la mucosa intestinal provocando fiebre y diarrea que puede llegar a ser tipo disentería

How well did you know this?
1
Not at all
2
3
4
5
Perfectly
96
Q

son cuadros en los que la clínica sistémica predomina sobre la digestiva: fiebre, cefalea, leucopenia sin eosinofilia, dolor abdominal, esplenomegalia y bradicardia relativa. Se deben a bacterias que
penetran la mucosa intestinal intacta y desde ahí pasan al torrente circulatorio

A

Fiebres entéricas: Yersinia enterocolitica, Salmonella typhi y paratyphi.

How well did you know this?
1
Not at all
2
3
4
5
Perfectly
97
Q

es un cuadro de fiebre entérica que se puede acompañar de exantema macular en tórax y abdomen, que cede de forma espontánea en pocos días, así como alteraciones del nivel de consciencia.

A

fiebre tifoidea

roséola tifoidea

How well did you know this?
1
Not at all
2
3
4
5
Perfectly
98
Q

El diagnóstico de elección de la fiebre tifoidea es

A

el cultivo, ya sea mediante hemocultivo las primeras dos semanas (más rentable para el diagnóstico precoz) o cultivo de heces a partir de la 3era semana.

How well did you know this?
1
Not at all
2
3
4
5
Perfectly
99
Q

Las complicaciones más relevantes de la fiebre tifoidea son

A

perforación intestinal, alteraciones neurológicas, miocarditis, hepatitis con o sin insuficiencia hepática, CID/SUH o hemorragias.

How well did you know this?
1
Not at all
2
3
4
5
Perfectly
100
Q

El tratamiento de la FIEBRE TIFOIDEA es

A

fluoroquinolonas o las cefalosporinas de tercera

generación.

How well did you know this?
1
Not at all
2
3
4
5
Perfectly
101
Q

Existe vacuna contra tifoidea con dos formas comerciales que son

A

Ty21 (oral) y polisacárido VI (parenteral).

How well did you know this?
1
Not at all
2
3
4
5
Perfectly
102
Q

a quienes se vacuna contra la tifoidea

A

La OMS recomienda la vacunación en mayores de 2 años que viven en zonas endémicas, viajeros a zonas
endémicas, personas en campos de refugiados, trabajadores de laboratorios clínicos y personas que
trabajen en desagüe.

How well did you know this?
1
Not at all
2
3
4
5
Perfectly
103
Q

DEFINICIÓN DE DIARREA

A

Expulsión de tres o más deposiciones líquidas en 24

horas.

How well did you know this?
1
Not at all
2
3
4
5
Perfectly
104
Q

DEFINICION DE EPISODIO DIARREICO

A

Diarrea (3 o más deposiciones liquidas en 24hrs) seguido de al menos 48hrs de deposiciones normales.

How well did you know this?
1
Not at all
2
3
4
5
Perfectly
105
Q

DURACION DE LA DIARREA AGUDA

A

<4 SEMANAS

How well did you know this?
1
Not at all
2
3
4
5
Perfectly
106
Q

DURACION DE LA DIARREA CRÓNICA

A

> 4 SEMANAS

How well did you know this?
1
Not at all
2
3
4
5
Perfectly
107
Q

La diarrea que resulta poco después de haber comido POLLO

A

Salmonella, Campylobacter o Shigella proveniente del pollo

How well did you know this?
1
Not at all
2
3
4
5
Perfectly
108
Q

La diarrea que resulta poco después de haber comido carne molida cruda;

A

E. coli enterohemorragica (O157:H7)

How well did you know this?
1
Not at all
2
3
4
5
Perfectly
109
Q

La diarrea que resulta poco después de haber comido arroz frito u otros alimentos recalentados

A

Bacillus cereus

How well did you know this?
1
Not at all
2
3
4
5
Perfectly
110
Q

La diarrea que resulta poco después de haber comido mayonesa o crema

A

Staphylococcus aureus o Salmonella

How well did you know this?
1
Not at all
2
3
4
5
Perfectly
111
Q

La diarrea que resulta poco después de haber comido HUEVOS CRUDOS

A

Salmonella

How well did you know this?
1
Not at all
2
3
4
5
Perfectly
112
Q

La diarrea que resulta poco después de haber comido alimentos crudos o quesos blandos

A

Listeria

How well did you know this?
1
Not at all
2
3
4
5
Perfectly
113
Q

La diarrea que resulta poco después de haber comido mariscos crudos

A

especies de Vibrio, Salmonella o hepatitis A

How well did you know this?
1
Not at all
2
3
4
5
Perfectly
114
Q

EN LAS INTOXICACIONES ALIMENTARIAS los síntomas empiezan a las

A

4 A 6HRS DESPUÉS DE LA INGESTA.

How well did you know this?
1
Not at all
2
3
4
5
Perfectly
115
Q

características normales de citología de moco fecal

A

NO suelen tener ni leucocitos, ni eritrocitos, ni células epiteliales. Pueden tener pocos leucocitos.
Presencia de células epiteliales pueden ser por irritación gastrointestinal.

How well did you know this?
1
Not at all
2
3
4
5
Perfectly
116
Q

características de la citología de moco fecal en pacientes con diarrea infecciosa

A

La LEUCOCITOSIS ABUNDANTE se encuentra

elevada en PMN

How well did you know this?
1
Not at all
2
3
4
5
Perfectly
117
Q

características de la citología de moco fecal en pacientes con intolerancia a lactosa.

A

PRESENCIA DE AZUCARES REDUCTORES EN HECES

How well did you know this?
1
Not at all
2
3
4
5
Perfectly
118
Q

TRATAMIENTO DE DIARREA NO INFLAMATORIA

A

HIDRATACION ES LA PIEDRA ANGULAR DEL TRATAMIENTO INDEPENDIENTE DE LA ETIOLOGIA DE LA DIARREA
Desde 2006 se recomienda las soluciones hipotónicas de la OMS que contienen una osmolaridad menor a la normal.

How well did you know this?
1
Not at all
2
3
4
5
Perfectly
119
Q

DIAGNOSTICO DE LA DIARREA INFLAMATORIA

A

REALIZAR COPROCULTIVO Y COPROPARASITOSCOPICO EN SERIE III AL PRESENTAR DATOS DE SRIS O DISENTERÍA

How well did you know this?
1
Not at all
2
3
4
5
Perfectly
120
Q

AZUL DE METILENO EN FRESCO, PARA DX DE DIAREA INFLAMATORIA ES POSITIVO CUANDO

A

cuando existe la presencia de más de 10 leucocitos por campo.

How well did you know this?
1
Not at all
2
3
4
5
Perfectly
121
Q

TRATAMIENTO DE LA DIARREA INFLAMATORIA

A

¡HIDRATACIÓN Y DIETA ASTRINGENTE ES LA PIEDRA ANGULAR, ES IMPORTANTE ATENCION A SIGNOS DE
DESHIDRATACIÓN EN PACIENTES ANCIANOS PUES ES ALTO EL RIESGO DE MUERTE!

How well did you know this?
1
Not at all
2
3
4
5
Perfectly
122
Q

INDICACION DE ANTIBIOTICOS EN DIARREA

A

en pacientes que presentan DISENTERÍA, estados de INMUNOSUPRESIÓN, ANCIANOS, VÁLVULA CARDÍACA.

How well did you know this?
1
Not at all
2
3
4
5
Perfectly
123
Q

TRATAMIENTO FARMACOLOGICO ESPECIFICO DE SHIGELLA

A

CIPROFLOXACINO 500MG 2 VECES AL DIA X 1-3 DIAS

How well did you know this?
1
Not at all
2
3
4
5
Perfectly
124
Q

TRATAMIENTO FARMACOLOGICO ESPECIFICO DE SALMONELA NO TYPHI

A

TMP-SML O CIPRO X5-7 DIAS

How well did you know this?
1
Not at all
2
3
4
5
Perfectly
125
Q

TRATAMIENTO FARMACOLOGICO ESPECIFICO DE E. COLI

A

CIPRO

How well did you know this?
1
Not at all
2
3
4
5
Perfectly
126
Q

TRATAMIENTO FARMACOLOGICO ESPECIFICO DE YERSENIA

A

DOXICILINIA + AG

How well did you know this?
1
Not at all
2
3
4
5
Perfectly
127
Q

TRATAMIENTO FARMACOLOGICO ESPECIFICO DE VIBRIO CHOLERAE

A

DU DE DOCI 300MG

TETRACICLINA 500MG 4 VECES AL DIA X3 DIAS

How well did you know this?
1
Not at all
2
3
4
5
Perfectly
128
Q

TRATAMIENTO FARMACOLOGICO ESPECIFICO DE C. DIFFICILLE, GIARDIA Y E. HISTOLYTICA

A

METRONIDAZOL 250MG 4 VECES AL DIA POR 10 DIAS

How well did you know this?
1
Not at all
2
3
4
5
Perfectly
129
Q

TRATAMIENTO FARMACOLOGICO ALTERNATIVO DE E. HISTOLYTICA

A

IODOQUINOL

How well did you know this?
1
Not at all
2
3
4
5
Perfectly
130
Q

TRATAMIENTO FARMACOLOGICO ESPECIFICO DE SALMONELLE THYPI

A

CIPROFLOXACINO

How well did you know this?
1
Not at all
2
3
4
5
Perfectly
131
Q

En el uso de la loperamida se han encontrado las siguientes contraindicaciones absolutas:

A

 Diarrea invasiva o inflamatoria.

 Colitis ulcerosa.

How well did you know this?
1
Not at all
2
3
4
5
Perfectly
132
Q

DIAGNÓSTICO DE La INFECCIÓN POR ROTAVIRUS (Diarrea pediátrica)

A

inmunoensayo (Rotatest o microscopía)

How well did you know this?
1
Not at all
2
3
4
5
Perfectly
133
Q

La INFECCIÓN POR CALICIVIRUS (Diarrea pediátrica) ocasiona una DIARREA MODERADA-LEVE asociado a ALIMENTOS COMO

A

PESCADOS Y MARISCOS. El diagnostico se hace por inmunoensayo.

How well did you know this?
1
Not at all
2
3
4
5
Perfectly
134
Q

MANIFESTACIONES CLINICAS DE LA INFECCION POR ENTAMOEBA.

A

90% Individuos portadores asintomáticos
10% Amebiasis invasiva con Colitis amebiana: disentería, pérdida de peso y fiebre, tenesmo y dolor abdominal.
1% Existe enfermedad extraintestinal (La más frecuente el absceso hepático amebiano)

How well did you know this?
1
Not at all
2
3
4
5
Perfectly
135
Q

CUADRO CLINICO TIPICO DE ENTAMOEBA

A

Dolor en los cuadrantes inferiores del abdomen y diarrea leve, seguidos de MEG, adelgazamiento y dolor difuso en los cuadrantes, diarrea con 10 a 12 evacuaciones al día, heces con sangre y moco principalmente.

How well did you know this?
1
Not at all
2
3
4
5
Perfectly
136
Q

ESTUDIO DIAGNOSTICO DE LA INFECCION POR ENTAMOEBA

A

COPROPARASITOSCOPICO y EXAMEN DE HECES FECALES

  1. Sangre oculta en heces POSITIVA
  2. Neutrófilos escasos en citología de moco fecal
  3. Quistes o trofozoitos de Entamoeba histolitica.
How well did you know this?
1
Not at all
2
3
4
5
Perfectly
137
Q

El DIAGNÓSTICO DEFINITIVO de COLITIS AMEBIANA se establece al

A

demostrar EN EXAMEN EN FRESCO DE HECES SERIE DE 3 la presencia de: TROFOZOITOS HEMATOFAGOS DE E. HISTOLYTICA

How well did you know this?
1
Not at all
2
3
4
5
Perfectly
138
Q

LA PRUEBA MÁS ESPECÍFICA PARA DX de COLITIS AMEBIANA desde el punto de vista clínico ES

A

EL TEST DE ELISA para detectar el antígeno de lectina

de adherencia de E. histolytica; LA SENSIBILIDAD ES SUPERIOR AL 85% Y SU ESPECIFICIDAD MAYOR DEL 90%.

How well did you know this?
1
Not at all
2
3
4
5
Perfectly
139
Q

El MÉTODO DIAGNÓSTICO GOLD ESTÁNDAR para Colitis amibiana es

A

COLONOSCOPIA CON TOMA DE BIOPSIAS

How well did you know this?
1
Not at all
2
3
4
5
Perfectly
140
Q

TRATAMIENTO DE LA AMEBOSIS INTESTINAL O COLITIS AMIBIANA en Pacientes portadores asintomáticos

A

Paromomicina o Yodoquinol 650mg c8hrs x 20 días

How well did you know this?
1
Not at all
2
3
4
5
Perfectly
141
Q

TRATAMIENTO DE LA AMEBOSIS INTESTINAL O COLITIS AMIBIANA en Pacientes con colitis aguda:

A

Metronidazol 750mg VO cada 8hrs x 5 días o tinidazol 2gr c24hrs VO por 3 días + Fármacos luminales para evitar recurrencias.
En caso de no contar con Metronidazol usar Eritromicina para enfermedad intestinal

How well did you know this?
1
Not at all
2
3
4
5
Perfectly
142
Q

UANDO SE ENCUENTRE SANGRE OCULTA EN HECES SIN LEUCOCITOSIS, SOSPECHAR

A

NEOPLASIAS O ISQUEMIA

How well did you know this?
1
Not at all
2
3
4
5
Perfectly
143
Q

Un paciente de 78 años, previamente sano, que vive en una residencia de ancianos bastante masificada y con insuficientes recursos higiénicos, padece un cuadro diarreico desde hace 6 semanas. Refiere molestias abdominales tipo retortijón, febrícula ocasional y 4-6 deposiciones diarias, alguna de ellas nocturna, con mucosidad y, en ocasiones, con hebras de sangre. Entre los diagnósticos que se enumeran a continuación, seleccione el que le parece MENOS probable:

1) Cáncer de colon.
2) Infección por Clostridium difficile.
3) Enfermedad de Crohn.
4) Salmonelosis

A

4

How well did you know this?
1
Not at all
2
3
4
5
Perfectly
144
Q

A 38-year-old male presented to the emergency room complaining of severe abdominal pain, fever of 38 ºC and dark-colored vomit dark. Also related was liquid diarrhea with 12 stool depositions a day. Fever since one month has gradually worsened. He also complains of arthralgia of the elbows, knees and wrists of two weeks clinical course. Physical examination revealed a distended abdomen and tympanic, with diffuse abdominal pain, and guarding. Analysis is performed showing: hemoglobin 10.2 g/dL, MCV 68 fl, WBC 25,100/mm3 with 89% neutrophils, platelets 280,000/mm3, glucose 100 mg/dl, urea 34 mg/dl, creatinine 1.6 mg/dl, amylase 69 U/ml, sodium 138, potassium 3.5. Based on the most likely diagnosis, which is the least (menos) suitable approach?

1) Order colonoscopy for diarrhea etiologic affiliation.
2) Stool cultures and parasites.
3) Commence antibiotic therapy with metronidazole.
4) Rehydration with oral solution.

A

1

How well did you know this?
1
Not at all
2
3
4
5
Perfectly
145
Q

DIARREA SECRETORA se caracterizan por

A

Heces de gran volumen, acuosas >1L al día PERSISTENTE CON EL AYUNO.

How well did you know this?
1
Not at all
2
3
4
5
Perfectly
146
Q

ETIOLOGIA DE DIARREA CRONICA SECRETORA MAS FRECUENTE

A

POR FÁRMACOS
Antibióticos, Medicamentos con Arsénico, anti-arrítmicos cardiacos, antihipertensivos, AINES, antidepresivos ISRS, anti-neoplásicos, broncodilatadores, antiácidos y laxantes.

How well did you know this?
1
Not at all
2
3
4
5
Perfectly
147
Q

Uno de los TUMORES NEUROENDOCRINOS MÁS FRECUENTES, se manifiesta casi siempre por ulceras pépticas resistentes, pero en un 30% produce diarrea y en un 10% este síntoma puede ser la única manifestación clínica. ESTE TUMOR SE LLAMA

A

SÍNDROME DE ZOLLINGER-ELLISON (Ulceras pépticas resistentes + Diarrea)
La DIARREA surge por digestión deficiente de las grasas porque el pH intraduodenal bajo inactiva las enzimas pancreáticas.

How well did you know this?
1
Not at all
2
3
4
5
Perfectly
148
Q

DIARREA OSMOTICA SE DEFINE POR

A

(>50mOsm/kg, Si hay GAP fecal) Está causada por el acúmulo de solutos no absorbibles en la luz intestinal.
Esta diarrea cesa cuando el paciente ayuna y el soluto gap del fluido fecal está aumentado (>50MoSM/kg)

How well did you know this?
1
Not at all
2
3
4
5
Perfectly
149
Q

DIFERENCIA ENTRE DIARREA SECRETORA Y OSMOTICA

A

la diarrea OSMOTICA CEDE EN EL AYUNO, LA SECRETORA NO

How well did you know this?
1
Not at all
2
3
4
5
Perfectly
150
Q

Una de las CAUSAS MÁS COMUNES DE DIARREA CRÓNICA en adultos es la

A

EFICIENCIA DE LACTASA, que afecta a 75% de personas.

How well did you know this?
1
Not at all
2
3
4
5
Perfectly
151
Q

LA INTOLERANCIA AL GLUTEN SE CARACTERIZA POR

A

diarrea crónica, distensión y dolor abdominal son síntomas de la intolerancia al gluten no celiaca e intolerancia a los carbohidratos fermentables (FODMAP)

How well did you know this?
1
Not at all
2
3
4
5
Perfectly
152
Q

pH bajo en las heces puede indicar

A

malabsorción de carbohidratos. La más común: Intolerancia lactosa

How well did you know this?
1
Not at all
2
3
4
5
Perfectly
153
Q

La esteatorrea se define como

A

la cantidad de grasa en heces que rebasa los 7 g/día

How well did you know this?
1
Not at all
2
3
4
5
Perfectly
154
Q

CAUSAS DE ESTEATORREA

A

 Mala digestión intraluminal por insuficiencia exocrina del páncreas debido a pancreatitis crónica o fibrosis quística.
 Cirrosis o la obstrucción biliar pueden causar esteatorrea leve al disminuir la concentración
intraluminal de los ácidos biliares.
 Malabsorción de la mucosa por Enfermedad infecciosa como Giardia o Enfermedad celiaca por
atrofia de vellosidades. (1% de la población)
 Enfermedad de Whipple: Infiltración de células histiociticas de la mucosa del intestino delgado con
fiebre, artralgias y fatiga externa acompañado de esteatorrea.

How well did you know this?
1
Not at all
2
3
4
5
Perfectly
155
Q

Alteración en la coloración del colon por ingesta crónica de laxantes en la colonoscopia

A

melanosis coli EN DIARREA FACTICIA O POR ABUSO DE LAXANTES

How well did you know this?
1
Not at all
2
3
4
5
Perfectly
156
Q

es una infección localizada que afecta al tejido celular subcutáneo y a la fascia subyacente sin afectar planos musculares. Entre los agentes causales más comunes se encuentran SBHGA, S. aureus, Clostridium.

A

CELULITIS

How well did you know this?
1
Not at all
2
3
4
5
Perfectly
157
Q

Es una infección rápidamente progresiva que afecta piel, tejido celular subcutáneo, fascia superficial y profunda.

A

Fascitis necrotizante

How well did you know this?
1
Not at all
2
3
4
5
Perfectly
158
Q

ETIOLOGIA DE LA Fascitis necrotizante

A

POLIMICROBIANA MIXTA CON ANAEROBIOS

How well did you know this?
1
Not at all
2
3
4
5
Perfectly
159
Q

CLINICA DE LA Fascitis necrotizante

A
  1. Signos locales: de forma progresiva aparece dolor intenso localizado, eritema y edema extenso,
    equimosis y áreas de anestesia cutánea, ampollas y vesículas (bullas), presencia de gas en los tejidos y
    extensión de la clínica por espacio de fascias, puede llegar a producir miositis con aumento de niveles
    de CPK.
  2. Signos sistémicos: puede evolucionar a necrosis tisular y a sepsis o choque séptico. Cuando esto ocurre aparecen las alteraciones generales.
How well did you know this?
1
Not at all
2
3
4
5
Perfectly
160
Q

En las fases iniciales de la FASCITIS NECROZANTE es característica

A

La disociación entre el dolor intenso que refiere el enfermo y la escasa afectación cutánea que se objetiva.

How well did you know this?
1
Not at all
2
3
4
5
Perfectly
161
Q

fascitis necrotizante con localización escrotal y

perineal, producida por una afección mixta.

A

gangrena de Fouriner

How well did you know this?
1
Not at all
2
3
4
5
Perfectly
162
Q

ETIOLOGIA DE LA Gangrena gaseosa

A

C. perfingens, son bacilos

grampositivos anaerobios, encapsulados y esporulados

How well did you know this?
1
Not at all
2
3
4
5
Perfectly
163
Q

ANTECEDENTE DE IMPORTANCIA EN PACIENTES CON GANGRENA GASEOSA

A

antecedente de traumatismo evidente (heridas sucias y penetrantes con lesión vascular).
Es característico el intenso dolor.
La infección con frecuencia queda localizada en el músculo. Suele existir edema y exudación de la herida, el gas aparece en fases más tardías

How well did you know this?
1
Not at all
2
3
4
5
Perfectly
164
Q

TRATAMIENTO DE LA GANGRENA GASEOSA

A

desbridamiento y antibioticoterapia con penicilina G asociada a clindamicina.

How well did you know this?
1
Not at all
2
3
4
5
Perfectly
165
Q

principales gérmenes asociados a Mordedura de perro son

A

Pasteurella, S. aureus y estreptococos. Generalmente se asocian a celulitis alrededor de la herida.
Sin aparece sepsis se debe sospechar Capnocytophaga canimorsus.

How well did you know this?
1
Not at all
2
3
4
5
Perfectly
166
Q

Arañazo de gato: suele estar causada por

A

Bartonella henselae. Es más frecuente en niños y suele

producir una lesión papulocostrosa con linfadenopatía regional autolimitada en 3-6 semanas.

How well did you know this?
1
Not at all
2
3
4
5
Perfectly
167
Q

CUADRO CLINICO DE INFECCION POR YERSINIA ENTEROCOLÍTICA

A

CUADRO DIARREICO CARACTERIZADO POR FIEBRE Y DOLOR EN FID DEBIDO A INFLAMACIÓN DE LAS PLACAS DE PEYER ACOMPAÑADO DE ODINOFAGIA, DANDO UN CUADRO DE PSEUDOAPENDICITIS O SINDROME PSEUDOAPENDICULAR.

How well did you know this?
1
Not at all
2
3
4
5
Perfectly
168
Q

DX DE INFECCION POR YERSINIA ENTEROCOLÍTICA

A

PSEUDOAPENDICITIS + ILEITIS + LINFADENOPATÍA

MESENTÉRICA

How well did you know this?
1
Not at all
2
3
4
5
Perfectly
169
Q

TTO PARA INFECCION POR YERSINIA ENTEROCOLÍTICA

A

NO EXISTE TRATAMIENTO ANTIBIOTICO RECOMENDADO PARA YERSINIA EN FASE AGUDA

How well did you know this?
1
Not at all
2
3
4
5
Perfectly
170
Q

VIA DE TRANSMISIÓN DE LA FIEBRE TIFOIDEA.

A

El ser humano es el único reservorio de SALMONELLA TYPHI Y PARATYPHI.
El modo de transmisión más común de la fiebre tifoidea es a través del agua y los alimentos contaminados con heces u orina

How well did you know this?
1
Not at all
2
3
4
5
Perfectly
171
Q

DX DE FIEBRE TIFOIDEA

A
FIEBRE (PRINCIPAL) 
CEFALEA Y MALESTAR 
MIALGIAS Y ARTRALGIAS 
HEPATOESPLENOMEGALIA
ROSEOLA TIFOIDEA (EXANTEMA) 
PERFORACION Y HEMORRAGIA INTESTINAL
How well did you know this?
1
Not at all
2
3
4
5
Perfectly
172
Q

MÉTODO MÁS FIABLE para el diagnóstico precoz DE FIEBRE TIFOIDEA

A

es el CULTIVO, con mayor rentabilidad en hemocultivos en la primera semana o mediante el CULTIVO DE LAS HECES a partir de la TERCERA SEMANA.

How well did you know this?
1
Not at all
2
3
4
5
Perfectly
173
Q

CUANDO SE DEBE SOLICITAR LA AGLUTINACIÓN DE WIDAL O REACCIONES FEBRILES PARA FIEBRE TIFOIDEA

A

SOLICITAR A PARTIR DE LA SEGUNDA SEMANA Y SE CONSIDERARÁ POSITIVO CUANDO TITULOS O Y H >1:160

How well did you know this?
1
Not at all
2
3
4
5
Perfectly
174
Q

Se recomienda que a todo paciente con SÍNDROME FEBRIL DE MÁS DE UNA SEMANA DE EVOLUCIÓN con reacción de Widal positiva (TÍTULOS DE ANTÍGENOS O Y H >1:160) se le realicen

A

hemocultivo, mielocultivo, urocultivo y coprocultivo

How well did you know this?
1
Not at all
2
3
4
5
Perfectly
175
Q

ESTANDAR DE ORO DE FIEBRE TIFOIDEA

A

MIELOCULTIVO

176
Q

COMPLICACIONES DE LA FIEBRE TIFOIDEA

A
PERFORACION Y HEMORRAGIA  INTESTINAL
ALTERACIONES NEUROLÓGICAS 
MIOCARDITIS
 HEPATITIS 
CID
177
Q

TRATAMIENTO DE ELECCION PARA FIEBRE TIFOIDEA

A
  1. AMPICILINA el de elección
178
Q

TRATAMIENTO DE ELECCION EN CASO DE BROTES DE FIEBRE TIFOIDEA

A
  1. AZITROMICINA el de elección en caso de brotes
179
Q

TRATAMIENTO en caso de falla ambulatorio y resistencia a antibióticos o Fiebre tifoidea
complicada

A
  1. CEFALOSPORINAS DE 3ERA
180
Q

Presencia de brotes o epidemias de cualquier

enfermedad, se considera una notificación de tipo

A

INMEDIATA <24 HORAS

181
Q

: Evento que, por su magnitud o trascendencia, requiere la inmediata instrumentación de acciones.

A

URGENCIA EPIDEMIOLÓGICA

182
Q

Al evento de nueva aparición o reaparición, cuya presencia pone en riesgo la salud de la población, y que por su magnitud requiere de acciones inmediatas.

A

EMERGENCIA EPIDEMIOLÓGICA

183
Q

El término “meningitis aséptica” se aplica a aquellas meningitis, habitualmente linfocitarias, producidas por

A

enterovirus

184
Q

etiología de Meningitis mas frecuente en el mundo

A

neumococo y el meningococo, seguidos por Listeria y

estafilococos

185
Q

etiología de Meningitis frecuente en RN

A

SBHGA (agalactiae), bacilos gramnegativos y listeria.

186
Q

etiología de Meningitis frecuente en niños y adultos jóvenes

A

meningococo, neumococo y Haemophilus influenzae tipo b.

187
Q

es la causa más frecuente de meningitis secundaria a fístula del LCR (por fractura de la base del cráneo) y de meningitis recurrente. Los esplenectomizados, hipogammaglobulinemia y alcohólicos tienen especial susceptibilidad.

A

Neumococo

188
Q

Las meningitis virales son cuadros de

A

fiebre, cefalea, escasa rigidez de nuca

189
Q

Las meningitis bacterianas cuentan con CLINICA de

A

rigidez marcada, signos meníngeos positivos, fiebre elevada, náuseas y vómitos, sudoración y postración. Hay afectación de nervios craneales, fotofobia, confusión o convulsiones.

190
Q

en meningitis con xantema maculoeritematoso diseminado o insuficiencia suprarrenal aguda por necrosis hemorrágica de las suprarrenales, la etiología es por

A

meningococo

191
Q

En caso de meningitis virales, lo característico del LCR es

A

Un moderado número de células (<1,000) con predominio de mononucleares, glucosa normal y proteínas normales o levemente aumentadas.

192
Q

En caso de meningitis BACTERIANAS, lo característico del LCR es

A

(purulentas) hay aumento de PMN, glucosa disminuida (correlacionarla con la central) y las proteínas elevadas. La tinción Gram y el cultivo ayudan a la orientación etiológica.

193
Q

En las meningitis bacteriana el tratamiento de elección son

A

cefalosporinas de tercera generación o el meropenem.

Si se sospecha resistencia a penicilina se debe añadir vancomicina.

194
Q

En pacientes neuroquirúrgicos con derivación de LCR o con TCE, se debe iniciar tratamiento empírico para meningitis, con

A

vancomicina y cefepime

195
Q

En niños se ha demostrado que la administración de esteroides disminuye la incidencia de complicaciones por meningitis, al disminuir la inflamación meníngea. El esteroide de elección es

A

dexametasona a 0.15 mg/kg.

196
Q

La profilaxis en contactos cercanos de un enfermo con sospecha de meningitis menigocócica se realiza con

A

rifampicina 600 mg cada 12 por 2 días o
ciprofloxacino dosis única o
ceftriaxona 250 mg a 1 g intramuscular dosis única.

197
Q

Es la forma más frecuente de encefalitis esporádica en adultos inmunocompetentes

A

Encefalitis por virus del herpes simple 1

198
Q

En la encefalitis por virus del herpes simple 1, el LCR se encuentra

A

con aumento de linfocitos y proteínas con glucosa normal, así como presencia de eritrocitos hasta en 20% de los casos.

199
Q

dx definitivo de la encefalitis por virus del herpes simple 1

A

con demostración del ADN del virus mediante PCR

200
Q

tétanos es producido por la exotoxina de Clostridium tetani, denominada

A

tetanoespasmina

201
Q

El cuadro clínico de tetanos comienza con

A

cefalea, irritabilidad y rigidez muscular, tras una incubación de dos semanas.
El período de estado se caracteriza por la presencia de trismo, risa sardónica, posición en opistótonos y espasmos que pueden afectar a las extremidades, musculatura respiratoria o laríngea.
Se acompaña de alteraciones vegetativas como fiebre, diaforesis, taquicardia, hipertensión/hipotensión.
Se conserva el nivel de consciencia en todo momento.

202
Q

El tratamiento de tentanos consiste en

A

medidas de soporte sin estímulos visuales ni auditivos, garantizando adecuada ventilación e hidratación y tratando precozmente las complicaciones.
Se deme administrar gammaglobulina antitetánica y metronidazol o penicilina.

203
Q

Botulismo: es producido por

A

la toxina botulínica de Clostridium botulinum

204
Q

la toxina botulínica de Clostridium botulinum actúa a nivel

A

presináptico inhibiendo la liberación de acetilcolina, y con ello impidiendo la contracción del músculo y dando lugar al cuadro de parálisis motora que caracteriza al botulismo

205
Q

Existen 8 serotipos de toxina botulinica, cuales son los que afectan al humano y cual es el más grave

A

los A, B y E afectan al humano y el A el más grave

206
Q

clinica de infeccion por Clostridium botulinum,

A

comenzar con síntomas digestivos, que siguen de afectación neurológica que comienza por los nervios más cortos, inicialmente con nervios craneales altos (diplopía y midriasis), posteriormente bajos y
finalmente nervios periféricos.

207
Q

diagnóstico de Clostridium botulinum,

A

se realiza con la clínica y mediante el aislamiento de
la toxina en sangre, heces, heridas o alimentos.
El LCR es normal.

208
Q

tratamiento del Clostridium botulinum,

A

es de sostén, desbridamiento de la herida, aceleración del tránsito intestinal para disminuir

209
Q

tiempo de incubación del virus de la rabia

A

1-3 meses

210
Q

patogenicidad del virus de la rabia es por

A

El virus se replica en las células musculares en el lugar de inoculación, asciende por nervios hasta el SNC, donde se replica en las neuronas (principalmente ganglios basales y tronco); a través de SNA
se extiende a numerosos tejidos.

211
Q

fases de la infeccion por el virus de la rabia

A

-fase prodrómica inespecífica
-fase de encefalitis aguda similar a la producida por otros virus
-fase con alteración del tronco del encéfalo que da
lugar a la clínica típica de la encefalitis rábica: hipersalivación y disfagia (cuadro clásico de hidrofobia),
diplopía, espasmo laríngeo, alteraciones autonómicas cardiovasculares y como fase final la muerte.

212
Q

La confirmación de infeccion por virus de la Rabia se realiza con la autopsia, demostrando

A

presencia de cuerpos de Negri en el cerebro. El historial de exposición es el más importante en el
diagnóstico.

213
Q

La profilaxis en personas expuestas al virus de la rabia se realiza mediante

A

la administración de gammaglobulina antirrábica

humana y tres dosis de la vacuna

214
Q

QUIMIOPROFILAXIS EN CONTACTOS CERCANOS DE MENINGITIS BACTERIANA POR MENINGOCOCO

A
  1. Rifampicina 600mg cada 12 horas por 2 días (4D)
  2. Ciprofloxacino 500mg VO DU
  3. Ceftriaxona 250mg a 1gr IM DU
215
Q

DIAGNÓSTICO CLINICO DE MENINGITIS

A

PRESENCIA DE FIEBRE + RIGIDEZ DE CUELLO + ALTERACIONES DEL ESTADO MENTAL

216
Q

En la meningitis bacteriana adquirida en la
comunidad, se encontraron los siguientes signos y
síntomas:

A
  • CEFALEA (87%),
  • RIGIDEZ DE CUELLO (83%),
  • FIEBRE >38ºC (77%)
  • ALTERACIONES DEL ESTADO DE CONCIENCIA (ESCALA DE COMA DE GLASGOW ≤14 PUNTOS)
217
Q

LIQUIDO CEFALORRAQUIDEO EN MENINGITIS BACTERIANA

A

Aspecto LCR turbio o purulento
Leucocitos en LCR: 1000 a 5000 células/mm3 (100-10,000)
Proporción leucocitos: 90% polimorfonucleares y 10% de mononucleares
Hipoglucorraquia <40mg/dl
Hiperproteirraquia >100mg/dL
PUNCION LUMBAR TRAUMÁTICA. >500 eritrocitos/mm3

218
Q

EL ESTUDIO MÁS SENSIBLE Y ESPECÍFICA DE MENINGITIS ES

A

PROCALCITONINA >0.2NG/ML PARA MENINGITIS

219
Q

ETIOLOGIA DE MENINGITIS EN IC, embarazo, puerperio, cáncer, EDAD >55 AÑOS, DM2

A

Listeria monocytogenes

220
Q

ETIOLOGIA DE MENINGITIS EN Epidemias

A

Neisseria meningitidis

221
Q

ETIOLOGIA DE MENINGITIS EN Heridas

A

Staphylococcus aureus + Streptococcus pneumoniae

222
Q

ETIOLOGIA DE MENINGITIS EN Válvulas VP

A

Staphylococcus epidermidis o aureus

223
Q

ETIOLOGIA DE MENINGITIS EN Postoperatoria

A

Gram-negativos + Pseudomonas aeruginosa

224
Q

TRATAMIENTO DE MENINGITIS BACTERIANA AGUDA EMPÍRICO EN INMUNOCOMPETENTE

A
  1. Cefalosporina 3era generación (Cefotaxima)
  2. Cloranfenicol (En caso de alergia a B-lactámicos) c/6hrs
  3. Meropenem
225
Q

TRATAMIENTO DE MENINGITIS BACTERIANA AGUDA POR NEUMOCOCO

A
  1. Penicilina (en caso de sensibilidad)
  2. Cefalosporina 3era (en caso de resistencia)
  3. Vancomicina
226
Q

TRATAMIENTO DE MENINGITIS BACTERIANA AGUDA POR LISTERIA

A
  1. Ampicilina 2gr IV c 4 horas + Gentamicina

2. Amoxicilina 2gr IV c 4 horas

227
Q

USO DE DEXAMETASONA , Se indica en caso de MENINGITIS POR

A

NEUMOCOCICA O POR HiB.
En sospecha o diagnóstico de meningitis bacteriana neumocócica puede estar indicada la aplicación de
dexametasona 0.15mg/kg cada 6 horas durante 4 días con la primera dosis administrada 10 minutos antes de la primera dosis de antimicrobiano.
Se recomienda la adición de rifampicina o vancomicina junto con la Cefalosporina en pacientes con meningitis neumocócica.

228
Q

En caso de presentarse con crisis convulsivas, en meningitis aguda el tratamiento es con

A

FENITOINA INTRAVENOSA es el anticomicial de elección

229
Q

Todos los pacientes con infección meníngea aguda con riesgos tromboembólicos deben recibir tratamiento Tromboprofiláctico con

A

Heparina NF 5000UI c 12hrs

230
Q

Reaparición de signos con un cultivo de LCR (+) durante el tratamiento de meningitis aguda, después de una buena respuesta al tratamiento inicial y esterilización del LCR.

A

RECRUDESCENCIA

231
Q

Nuevo episodio de MB en el periodo de convalecencia por sobreinfección de un germen diferente.

A

RECURRENCIA

232
Q

Presencia de signos y síntomas en MB, sí como laboratorio dentro de 3 semanas posteriores de haber terminado el esquema antibiótico. Reaparece el mismo
germen del evento primario

A

RECAÍDA

233
Q

Presenta un cuadro persistente que incluye CEFALEA, RIGIDEZ DE CUELLO, FEBRÍCULA Y LETARGIA DÍAS
O SEMANAS antes de acudir al médico para evaluación.

A

MENINGITIS SUBAGUDA

234
Q

ETIOLOGIA DE LA MENINGITIS SUBAGUDA Y CRÓNICA

A
M. tuberculosis 
H. capsulatum
C. immitis 
C. neoformans 
T. pallidum
235
Q

meningitis subaguda que puede ocasionar una ARACNOIDITIS en EL ESPACIO SUBARACNOIDEO

A

MENINGITIS POR MYCOBACTERIUM TUBERCULOSIS

causando una reacción inflamatoria intensa en la base del cráneo y que rodea los pares craneales.

236
Q

El diagnóstico de meningitis sifilítica se confirma con

A

algún método treponemico reactivo en suero (FTA-ABS o MHA-TP) + Pleocitosis de linfocitos + Hiperproteirraquia.
La negatividad de FTA-ABS DESCARTA NEUROSIFILIS pero la Positividad FTA-ABS no la confirma pues puede ser
FALSO POSITIVO POR CONTAMINACIÓN CON SANGRE.

237
Q

Los principales microorganismos involucrados en la meningitis viral son:

A

 Enterovirus: Principal agente patógeno causante de la meningitis viral
 Virus varicela-zóster
 Herpes-virus tipo 2 más que el HSV-1

238
Q
Joven de 15 años previamente sano que acude a consulta por cefalea, fiebre de 39 grados y somnolencia, Usted le encuentra rigidez de nuca y signos meníngeos. ¿Cuál es su conducta inmediata? 
A.- Iniciar antibióticos 
B.- Tomar hemocultivo y punción lumbar 
C.- Indicar una TAC 
D.- Indicar una RMN 
E.- Todos los anteriores.
A

b

239
Q

Joven de 15 años previamente sano que acude a consulta por cefalea, fiebre de 39° y somnolencia, Usted le encuentra rigidez de nuca y signos meníngeos. La enfermera le comunica que tiene unas ronchas en brazos y abdomen. Usted corrobora que se
trata de lesiones purpuricas. Su conducta inmediata es:
A.- Continuar con el tratamiento antibiótico
B.- Agregar dexametasona
C.- Indicar ciprofloxacina a los contactos cercanos
D.- Indicar rifampicina a los contactos cercanos
E.- A, C, D son correctas

A

e

240
Q

Joven de 15 años previamente sano que acude a consulta por cefalea, fiebre de 39° y somnolencia, Usted le encuentra rigidez de nuca y signos meníngeos. La enfermera le comunica que tiene unas ronchas en brazos y abdomen. Usted corrobora que se
trata de lesiones purpuricas. cual es el tratamiento de elección?
A.- Ceftriaxona mas vancomicina
B.- Penicilina o ceftriaxona o cefotaxima
C.- Cloranfenicol o Rifampicina
D.- Rifampicina más ciprofloxacina
E.- Ninguno de los anteriores.

A

b

241
Q
Diabética de 65 años que acude a consulta por crisis convulsivas, fiebre de 38 grados, vómito y fotofobia, Usted le encuentra signos meníngeos. El laboratorio le reporta una tinción de gram con formas cocobacilares gram positivas. Cuál es el tratamiento de elección para esta paciente? 
A.- Vancomicina más ceftriaxona 
B.- Cloranfenicol más rifampicina 
C.- Vancomicina 
D.- Ampicilina más gentamicina 
E.- Ninguno de los anteriores.
A

d

242
Q
Diabética de 65 años que acude a consulta por crisis convulsivas, fiebre de 38 grados, vómito y fotofobia, Usted le encuentra signos meníngeos. El laboratorio le reporta una tinción de gram con formas cocobacilares gram positivas. Cuál es el tratamiento de elección para esta paciente? 
A.- Vancomicina más ceftriaxona 
B.- Cloranfenicol más rifampicina 
C.- Vancomicina 
D.- Ampicilina más gentamicina 
E.- Ninguno de los anteriores.
A

d

243
Q

el DENGUE es causado por

A

arbovirus perteneciente a la familia Flaviviridae de los

Flavivirus compuesto por RNA de una sola cadena

244
Q

Es la infección más común transmitida por artrópodos a humanos y es endémica

A

DENGUE

245
Q

DENGUE se transmite por

A

la picadura del Aedes aegypti donde adquieren al virus
al picar en un hospedero, donde son capaces de transmitir la infección.
La transmisión por sangre infectada órganos o tejidos es posible por lo que la transmisión perinatal es factible.

246
Q

Periodo de incubación del DENGUE

A

5-7 DÍAS

247
Q

Fase febril DEL DENGUE

A
(2 a 5 días): 
 Fiebre >38.5°C 
 Mialgias y artralgias 
 Cefalea intensa 
 Leucopenia 
 Exantema maculopapular escarlatiniforme 
 Nausea y vómito
248
Q

Fase de defervescencia DEL DENGUE

A
(12 a 24hrs): Causada por extravasación del plasma al terminar la fiebre con un periodo de 24 a 48hrs 
aumentando la permeabilidad vascular. 
 Choque hipovolémico 
 Hipotensión arterial 
 Leucopenia 
 Derrame pleural o ascitis 
 Plaquetopenia <100,000 
 Elevación de AST y ALT 
 Aumento de hematocrito o disminución en casos de 
hemorragia grave 
 Hemorragias masivas 
 CID
249
Q

DATOS DE ALARMA DEL DENGUE

A
  • DOLOR ABDOMINAL O HIPERESTESIA
  • VOMITO PERSISTENTE
  • DERRAME PLEURAL
  • SANGRADO DE MUCOSAS
  • HMG >2CM
  • AUMENTO DEL HEMATOCRITO
  • DISNMUNCIÓN PLAQUETARIA
250
Q

Fase de recuperación DEL DENGUE

A

(>7 días): Existe una absorción de líquido, el estado hemodinámico se estabiliza, pueden tener un exantema tardío con descamación.
 Recuperación del hematocrito
 Aumento leucocitario normal
 Recuperación plaquetaria

251
Q

DIAGNOSTICO DEL DENGUE

A

Detección de IgM e IgG por ELISA: Considerado el estudio de elección para confirmar el diagnóstico y se eleva de 3 a 4 días posteriores a la fiebre.
Si el resultado es positivo se confirma el caso, si es negativo se debe realizar IgG

252
Q

TRATAMIENTO DE DENGUE

A
  1. Ingesta abundante de líquidos y tolerancia a la vía oral además de uso de antipiréticos (Paracetamol)
  2. En caso de signos de alarma, embarazadas, alto riesgo: Hidratación IV vigilando hematocrito que no aumente.
253
Q

TRATAMIENTO ANTE EXPOSICION POR RABIA

A
  1. Lavado de herida con agua y jabón y solución salina. Solución antiséptica.
  2. Aplicación de vacuna antitetánica
  3. Aplicación de vacuna antirrábica en caso de exposición de riesgo
  4. Vacunación en los días 0-3-7-14-28
254
Q

CLINICA DE Infección gonocócica EN HOMBRES

A

uretritis, que cursa con disuria y secreción uretral blanquecina escasa, de predominio matutino.
La clínica comienza de 2-5 días después de la exposición.

255
Q

CLINICA DE Infección gonocócica EN MUJERES

A

puede producir uretritis (síndrome miccional con urocultivo negativo) o cervicitis no complicada.
En caso de progresión puede dar lugar a endometritis, salpingitis, EPI, abscesos anexiales, peritonitis generalizada o de localización perihepática (síndrome de Fitz-Hugh-Curtis).

256
Q

La infección gonocócica diseminada, desencadenada frecuentemente en embarazo o menstruación, se
manifiesta como un cuadro de

A

fiebre, tenosinovitis y poliartralgias, con lesiones cutáneas papulares que se pueden hacer pustulosas o hemorrágicas, situadas característicamente sobre las articulaciones, típicamente monoarticular y de grandes articulaciones (rodilla, tobillo, muñeca)

257
Q

el diagnóstico de Infección gonocócica se realiza mediante

A

visualizando en la tinción de Gram el gonococo intracelular, en medios de cultivo específicos (Thayer-Martin) o bien mediante técnicas modernas de PCR.
En la infección diseminada los hemocultivos suelen ser positivos.

258
Q

el tratamiento de Infección gonocócica es

A

cefalosporina de 3era generación, como ceftriaxona IM (en dosis única, en caso de infección genital) o cefixima oral (DU). Son alternativas válidas una DU de ciprofloxacino oral o de azitromicina.

259
Q

En pacientes diagnosticados de infección gonocócica se debe realizar tratamiento empírico contra

A

Chlamydia,
ya que frecuentemente las infecciones van asociadas, y si no se trata esta última tarda en manifestarse; se
debe añadir azitromicina 2 g en dosis única.

260
Q

bacteria gramnegativa de crecimiento intracelular
obligado. Clínicamente produce cuadros de uretritis en ambos sexos y, en la mujer, cervicitis, endometritis,
salpingitis, EIP, peritonitis y perihepatitis

A

Chlamydia trachomatis

261
Q

diagnóstico de Chlamydia trachomatis

A

mediante tinción de Giemsa o técnicas de inmunofluorescencia directa en los exudados

262
Q

tratamiento de de Chlamydia trachomatis

A

doxiciclina vía oral durante 7-10 días o la dosis única de 1g de azitromicina.

263
Q

C. trachomatis también produce conjuntivitis en el recién nacido y los serotipos L1, L2 y L3 producen

A

linfogranuloma venéreo -> linfadenopatías inguinales con tendencia a la fistulización y la posterior
cicatrización espontánea a lo largo de varios meses. Se trata con doxiciclina 100 mg cada 12 horas por 7-
10 días.

264
Q

clínica del linfogranuloma venéreo

A

linfadenopatías inguinales con tendencia a la fistulización y la posterior cicatrización espontánea a lo largo de varios meses.

265
Q

tratamiento del linfogranuloma venéreo

A

doxiciclina 100 mg cada 12 horas por 7-10 días.

266
Q

Sífilis: es una ETS producida por

A

Treponema pallidum, bacteria perteneciente a las espiroquetas (forma de espiral, autopropulsión, anaerobias y no cultivables).

267
Q

PERIODO DE INCUBACION DE SIFILIS

A

21 DIAS

268
Q

Sífilis primaria: cuya lesión característica es

A

chancro duro, que aparece en el lugar de inoculación
(pene, vagina, ano, boca). Es una lesión sobreelevada, de consistencia cartilaginosa, no dolorosa, de fondo limpio, sin exudado y normalmente única.
Se acompaña de adenopatías regionales, normalmente
inguinales y bilaterales, duras, no dolorosas y no supurativas. Suele durar de 2-6 semanas.

269
Q

Sífilis secundaria: tras una fase asintomática de 6-8 semanas, aparece la clínica típica de la sífilis
secundaria que también dura de 2-6 semanas. Es una fase de generalización de la infección, caracterizada por

A

iebre, adenopatías, signos de afectación de diversos órganos (meningismo, artritis, hepatitis, neuritis, uveítis, nefropatía o gastritis) y las lesiones cutáneas características de esta fase: maculoeritematosas con afectación de palmas y plantas, leucoderma sifilítico (lesiones hipocrómicas en cuello; collarete de venus), zonas de foliculitis con alopecia parchada y la lesión característica, el condiloma plano, lesión muy infectiva en pliegues (submamario, inguinal, escrotal, axilar) en forma de placas no exudativas sobreelevadas

270
Q

Hasta el 30% de pacientes no tratados, al cabo de 20-30 años de la primoinfección, desarrollan sífilis
terciaria, cuya lesión cutánea característica es

A

la goma, lesión granulomatosa única o múltiple que
puede afectar a cualquier órgano. También pertenecen a sífilis terciaria los cuadros de afectación
cardiovascular con vasculitis y los cuadros de neurosífilis, meningitis subaguda o crónica y EVC

271
Q

Los cuadros de neurosífilis con afectación parenquimatosa son:

A

a. Tabes dorsal: cuadro de desmielinización de los cordones posteriores espinales que produce ataxia
sensitiva, que con el tiempo da lugar a lesiones cutáneas y deformidades articulares (principalmente
en miembro inferior).
b. Parálisis general progresiva: degeneración progresiva del SNC con alteraciones psiquiátricas,
motoras, intelectuales, de lenguaje y vegetativas (pupila de Argyll-Robertson).

272
Q

Diagnóstico DE SIFILIS

A
  • Visualización directa: inmunofluorescencia directa o microscopia de campo oscuro.
    -No se pueden cultivar.
  • Detección del material genético del germen: PCR.
  • Técnicas serológicas:
    a. Reagínicas: VDRL y RPR. Se emplean como cribado.
    b. Treponémicas: TPHA (microhemaglutinación) y FTA-abs (inmunofluorescencia directa con absorción
    de suero) que permiten la confirmación del diagnóstico.
273
Q

Sífilis primaria, secundaria y de latencia precoz se tratan con

A

penicilina G benzatínica en dosis IM única
de 2,4 millones de unidades.
Como alternativa: penicilina procaínica 600,000 UI por 10 días, ceftriaxona 500 mg por 10 días o
doxiciclina 100 mg cada 12 por 14 días.

274
Q

Sífilis latente tardía o de duración incierta, con LCR sin alteraciones, se trata con

A

penicilina G benzatínica en 3 dosis IM de 2.4 millones de unidades cada una.

275
Q

La neurosífilis se trata con

A

penicilina G acuosa IV durante 10-14 días.

276
Q

Reacción de Jarisch-Herxheimer: reacción ocasionada por

A

la liberación de endotoxinas por la lisis masiva
de espiroquetas, muy sensibles a la penicilina. Se manifiesta con fiebre, escalofríos, cefaleas, mialgia y
cuadros vegetativos. Se tarta de manera sintomática con antinflamatorios. En formas graves se trata
con esteroides.

277
Q

es una enfermedad de transmisión sexual producida por Haemophilus ducreyi, un coco bacilo gramnegativo

A

Chancro blando

278
Q

incubación del Chancro blando

A

de 3 días (chancro más precoz) se inicia la aparición de una lesión
blanda, pustulosa, no sobreelevada, dolorosa y con exudado que puede llegar a ser purulento.

279
Q

tratamiento para Chancro blando

A

ceftriaxona en dosis única.

280
Q

clinica del chanco blando

A

lesión blanda, pustulosa, no sobreelevada, dolorosa y con exudado que puede llegar a ser purulento.

281
Q

es la causa más frecuente de úlceras genitales. En el 70-90% de los casos es por

A

VHS-2.

282
Q

dx de VHS-2.

A

s clínico, mediante la visualización de las características células gigantes multinucleadas con inclusiones intracitoplasmáticas en el test de Tzank, o bien mediante técnicas de PCR.

283
Q

tratamiento de VHS-2.

A

aciclovir, Famciclovir o valaciclovir.

284
Q

Condiloma acuminado: producido por VPH serotipos

A

6 y 11

285
Q

Enfermedad de Lyme: es producida por

A

Borrelia burgdoferi, una espiroqueta gramnegativa de metabolismo anaerobio transmitida habitualmente por garrapatas del género Ixodes (garrapatas duras).

286
Q

Enfermedad de Lyme suele producirse en personas que realizan

A

cacería, acampadas o excursiones campestres.

287
Q

clínica de la Enfermedad de Lyme

A

aparición del típico eritema migratorio, que es una
mácula eritematosa con palidez central, frecuentemente indolora e iniciada en el lugar de picadura de la garrapata. Se suele localizar en ingle, muslo y axilas, y se resuelve de forma espontánea.

288
Q

síndrome de Bannwarth se asocia a

A

lesión de pares craneales, típicamente parálisis facial bilateral, manifestaciones oculares y cardíacas por infección diseminada de Borrelia burgdoferi,

289
Q

la Infección tardía persistente por Borrelia burgdoferi: puede aparecer meses o años después de la infección inicial. El cuadro típico consiste en

A

una artritis oligoarticular, de predominio en grandes articulaciones, principalmente rodillas. Puede haber signos cutáneos como acrodermatitis crónica atrófica.

290
Q

diagnóstico de Borrelia burgdoferi

A

s serológico por el difícil cultivo; entre las técnicas serológicas están la inmunofluorescencia indirecta y el ELISA. En la forma neurológica se necesitan anticuerpos de LCR superior
al sérico.

291
Q

tratamiento de de Borrelia burgdoferi

A

tetraciclina o amoxicilina (esta ultima en embarazadas o niños).
En los casos en los que existe lesión neurológica o articular grave, se aconseja utilizar ceftriaxona.
En afectación cardíaca se recomienda añadir esteroides.
Necesita vigilancia por posibilidad de reacción de Jarisch-Herxheimer.

292
Q

Leptospirosis: infección causada por

A

Leptospira interrogans, una espiroqueta de metabolismo aerobio

293
Q

la Leptospirosis Se transmite a partir de

A

animales domésticos y salvajes portadores o enfermos que eliminan el germen a través de la orina. El contagio puede ser por contacto directo con el animal o su orina, o indirectamente, sobre todo en el agua.

294
Q

cuadro clínico de Leptospirosis: Fase leptospirémica

A
  • presencia de leptospiras en sangre y LCR.
  • Comienza bruscamente con cefalea, mialgias (elevación de CPK), fiebre elevada y manifestaciones de diferentes órganos con fenómenos hemorrágicos. -Una forma grave es el síndrome de Weil (leptospirosis ictérica) que consiste en una lesión hepática con ictericia e insuficiencia renal. El signo clínico más característico de leptospirosis es la hemorragia conjuntival.
  • Tras 4-9 días la enfermedad mejora, coincidiendo con la desaparición del germen en sangre y LCR.
295
Q

cuadro clínico de Leptospirosis: Fase inmunitaria

A

coincide con la aparición de anticuerpos. La clínica es similar a la fase leptospirémica
y puede aparecer anemia hemolítica y leucocitosis importante. El LCR puede tener neutrofilia o aumento
de mononucleares

296
Q

tratamiento de de Leptospirosis

A

penicilina G. Como alternativas: tetraciclinas o eritromicina

297
Q

diagnóstico de de Leptospirosis

A

mediante cultivo en medios especiales en sangre o LCR en la primera fase y orina en la
segunda. Serología en la segunda fase.

298
Q

Carbunco: es producido por

A

Bacillus anthracis, un bacilo grampositivo, encapsulado, aerobio que forma colonias en forma de cabeza de medusa y endosporas. Produce una toxina responsable de un intenso
edema.

299
Q

el Carbunco Es una infección típica de

A

animales herbívoros que se produce en el hombre por contacto con animales infectados o sus productos.

300
Q

clínica del carbunco

A

más frecuente es la cutánea. Los pacientes presentan una lesión ulcerosa con una escara necrótica de color negruzco, típicamente indolora y rodeada por un intenso edema. El carbunco inhalado presenta como complicación una mediastinitis hemorrágica

301
Q

tratamiento del carbunco

A

penicilina. En bioterrorismo se recomienda ciprofloxacino.

302
Q

Tularemia: infección producida por

A

Francisella tularensis, bacilo gramnegativo aerobio que afecta a diversos animales

303
Q

Tularemia: Se transmite por

A

vector o contacto directo con liebres/conejos

304
Q

Tularemia:La infección suele adquirirse por

inoculación cutánea, de forma que la presentación es con

A

úlceras ganglionares es la más frecuente, consiste
en úlcera en sacabocados con adenopatías regionales. Menos frecuente es la orofaringea, neumónica y
tifoidea

305
Q

Tularemia:Se diagnostica por

A

serología

306
Q

Tularemia:el tratamiento consiste en

A

estreptomicina

307
Q

Peste: es producida por

A

Yersinia pestisis es un bacilo gramnegativo, anaerobio facultativo, inmóvil con una tinción bipolar en
imperdible

308
Q

Peste: Se transmite a través de

A

la picadura de la pulga de la rata o por contacto con animales, inhalación de material contaminado o de persona a persona en la forma neumónica.

309
Q

Peste: Tiene una forma clínica de

A

adenopática o bubónica de mayor frecuencia en la forma inguinal, una septicémica y otra neumónica.

310
Q

Peste: El

tratamiento de elección es

A

estreptomicina

311
Q

Déficit de IgA: el cuadro característico es la infección intestinal por

A

Giardia lamblia.

312
Q

Infecciones en receptores de trasplante:

- Primer mes:

A

infecciones nosocomiales y de neutropénico

313
Q

Infecciones en receptores de trasplante:

- 2°-sexto mes:

A

CMV causa más frecuente

314
Q

Infecciones en receptores de trasplante:

Más allá del sexto mes:

A

infecciones adquiridas en la comunidad.

315
Q

la brucelosis es una zoonosis producida por

A

el género Brucella, cocobacilos

gramnegativos aerobios de crecimiento intracelular facultativo

316
Q

Brucelosis o fiebre de Malta: hospedero habitual

A

animales. El hombre adquiere la infección de forma indirecta, tras la ingesta de productos lácteos
contaminados, como directa, tras el contacto con animales enfermos.

317
Q

Brucelosis o fiebre de Malta: CLINICA

A

es una enfermedad relativamente frecuente.
La clínica que produce es variable, tratándose de un
cuadro febril prolongado (patrón de fiebre continua ondulante) acompañado de sudoración profusa, astenia
y postración, cefalea, dolores articulares, HEM, adenopatías y otros síntomas diversos.
Además, produce una infección crónica localizada en diferentes sistemas, siendo los más destacados la
osteomielitis (vertebral), orquiepididimitis, meningoencefalitis, hepatitis granulomatosa y endocarditis sobre válvula aórtica,

318
Q

Brucelosis o fiebre de Malta: DX

A

mediante hemocultivo o mielocultivo, en el medio de Ruiz-Castañeda (crecimiento tardado de hasta 1 mes). La serología (Rosa de Bengala, test de Coombs) permite realizar un diagnóstico de presunción.
Títulos elevados de IgM indican exposición reciente y los de IgG enfermedad activa.

319
Q

Brucelosis o fiebre de Malta: Tratamiento

A

al menos seis semanas).
La combinación más eficaz es doxiciclina con AG (estreptomicina o gentamicina); otros útiles son las quinolonas y el cotrimoxazol.
En caso de meningoencefalitis y endocarditis debe administrarse doxiciclina + rifampicina + cotrimoxazol
por 6 meses añadiendo aminoglucósido el primer mes en caso de endocarditis.

320
Q

Nocardosis: CLÍNICA

A
  • Pulmonar: en forma de neumonía necrotizante o absceso pulmonar de evolución subaguda/oscilante.
  • SNC: ocasiona abscesos cerebrales.
321
Q

Nocardosis: DX

A

mediante la visualización de bacterias

filamentosas, que en su forma más característica son débilmente BAAR y se confirma mediante cultivo.

322
Q

Nocardosis: TTO

A

cotrimoxazol, durante 6-12 meses dependiendo la forma clínica. En el absceso cerebral se debe asociar ceftriaxona o Imipenem en ciclo corto.

323
Q

la actinomicosis es una infección crónica causada por

A

bacterias del género Actinomyces, las cuales son bacilos grampositivos, anaerobios, ramificados, no esporulados. Son gérmenes de baja patogenicidad. En su forma patógena afecta de manera típica a diversos tejidos, originando fibrosis, abscesos y fístulas.

324
Q

actinomicosis: CLÍNICA

A

formación de abscesos de evolución subaguda a
nivel de la región cervicofacial, torácica, abdominal o pélvica. En cualquiera de las localizaciones es
frecuente la tendencia a fistulizar hacia el exterior, drenando material purulento en forma de gránulos de
azufre (macrocolonias de Actinomyces).

325
Q

la actinomicosis es una de las pocas enfermedades infecciosas bacterianas que puede ser diagnosticada a través de

A

os hallazgos anatomopatológicos, aun en ausencia de cultivos positivos.
Toda secreción aspirada, tejido o fluidos o de absceso, debe ser estudiada a través de tinción de Gram,
observación en fresco y cultivos especiales. La tinción confirmatoria es Grocott-Gomory.

326
Q

TTO DE LA ACTINOMICOSIS

A

Las medidas no farmacológicas consisten en retiro de DIU y no permitir su uso por más de cinco años.
- tratamiento antibiótico de elección son los betalactámicos parenterales u orales.
El tiempo de duración debe ser prolongado de 2-6 semanas iniciando parenteral y posterior cambio a vía oral

327
Q

las principales rickettsiosis en México son

A

el tifo epidémico, tifo murino y la fiebre manchada de las montañas rocosas (R. ricketsii transmitida por Rhipicephalus sanguineus).

328
Q

Tifo endémico o murino es po:

A

pulga de la rata. Rickettsia typhi.

329
Q

Tifo epidémico es por

A

pulga de la rata. Rickettsia typhi.

330
Q

Enfermedad de Brill-Zinsser es por

A

reactivación tardía del tifo epidémico

331
Q

Fiebres manchadas y tifus: clinica

A

fiebre, malestar general, mialgias, cefalea

intensa y característicamente lesiones cutáneas eritematosas que afectan a palmas y plantas

332
Q

Fiebres manchadas y tifus: tto

A

serología.

-doxiciclina y se asocian esteroides en las formas graves

333
Q

La fiebre manchada de las montañas rocosas es una zoonosis producida por

A

Rickettsia rickettsii. El género
de Rickettsia tiene tropismo por el endotelio vascular; esto justifica manifestaciones graves como edemas,
hemorragias, LRA por hipovolemia, edema pulmonar no cardiogénico o encefalopatía por edema cerebral.

334
Q

fiebre manchada de las montañas rocosas: perido de incubacion

A

3-12 días posteriores a la mordedura de la garrapata inicia un cuadro clínico

335
Q

fiebre manchada de las montañas rocosas, cuadro clínico

A

ebre súbita, mialgias, cefalea y malestar general, asociados a dolor abdominal.
-La fase exantemática aparece entre el 3er-5to día de evolución.
-Los signos y síntomas tardíos (posterior al 3 días del inicio del cuadro) son el exantema, dolor abdominal y articular y el edema de manos y tobillos.
-El exantema inicialmente es centrípeto, macular, pequeño, no pruriginoso, palidece a la digitopresión, inicia en muñecas y tobillos e involucra palmas y plantas.
-Se torna papular y pueden aparecer petequias,
hemorragia franca en la piel y grandes áreas de necrosis e inclusive gangrena.

336
Q

fiebre manchada de las montañas rocosas, diagnóstico

A

examen físico del paciente y los datos epidemiológicos. El diagnóstico clínico es difícil por la inespecificidad de los síntomas iniciales.
Las pruebas serológicas tienen valor diagnóstico limitado ya que los anticuerpos son detectables hasta 7-10 días de inicio del cuadro (IFI es técnica estándar de oro).
Los hallazgos de laboratorio incluyen: anemia, trombocitopenia, hiperbilirrubinemia, aumento de transaminasas, elevación de CPK e hiponatremia.

337
Q

fiebre manchada de las montañas rocosas, tratamiento

A

doxiciclina
En >45 kg se utilizan 100 mg Vo cada 12 horas
en <45 kg 2.2 mg/kg.
Se puede administrar VO en pacientes que no vomiten o no estén deteriorados. En casos severos se pueden administrar hasta 4.4 mg/kg máximo de 200 cada 12 horas.
-La fiebre debe ceder las primeras 48 horas y se debe continuar el tratamiento por 7-10 días

338
Q

Fiebre Q: el agente causal es

A
Coxiella brunetii, que se transmite al ser humano por contacto directo con su 
hospedador habitual (vacas, ovejas o cabras) o por inhalación de esporas.
339
Q

Fiebre Q: clínica

A

a. Fase aguda: fiebre, astenia, cefalea y trombocitopenia y afectación pulmonar en forma de neumonía y lesión hepática con granulomas en forma de rosquilla.
b. Fase crónica: endocarditis que afecta de manera preferente a válvula aórtica con cultivos negativos.

340
Q

Fiebre Q: dx

A

es serológico con la peculiaridad de que la Coxiella tiene dos formas antigénicas, fase I y fase
II, que varían según el estadio de la enfermedad.
-El diagnóstico se confirma con la detección de anticuerpos de la fase II, mientras que en la crónica se detectan en la fase I

341
Q

Fiebre Q: tto

A

doxiciclina.

Se debe asociar levofloxacino o cotrimoxazol en la endocarditis.

342
Q

El virus JC (polyomavirus) causa

A

la Leucoencefalopatía multifocal progresiva

en sujetos inmunodeprimidos.

343
Q

El papiloma es causante de verrugas planas,

papilomas laríngeos y condilomas acuminados

A

tipo 6 y 11

344
Q

papiloma causante de cáncer de cérvix y ana

A

tipos 16 y 18

345
Q

papiloma causante de neoplasias nasales

A

16 y 57

346
Q

Es el agente etiológico de la mononucleosis infecciosa con anticuerpos heterófilos positivos. También se ha implicado en etiología de diversos tumores como carcinoma nasofaríngeo y linfoma de Burkitt.

A

virus de Epstein-Barr (VEB).

347
Q

Los signos de alarma del virus del dengue son:

A

dolor abdominal intenso, vómito persistente o incoercible, evidencia clínica de acumulación de líquidos, hemorragia activa en mucosas, alteraciones neurológicas o del estado de alerta y hepatomegalia/insuficiencia hepática.

348
Q

es el único ARV contraindicado durante el embrazo.

A

efavirenz

349
Q

habitualmente el diagnóstico de la infección VIH se realiza mediante

A

anticuerpos frente al VIH (serología). Para ello se emplean dos técnicas:

  1. ELISA: detecta anticuerpos frente a múltiples antígenos del VIH. Es una técnica muy sensible pero poco específica, de ahí que se emplea como cribado inicial.
  2. Western blot: detecta anticuerpos dirigidos exclusivamente frente a tres proteínas del VIH: gp41, gp120 y p24, apareciendo en forma de bandas con el peso molecular correspondiente a los productos génicos del VIH. Para que se considere positiva debe reconocer al menos 2 de esas bandas. Si solo se detecta 1 es indeterminable y se debe repetir en unas semanas.
350
Q

Clasificación de la infección por VIH: incluye la primoinfección clínica (síndrome retroviral agudo), la fase asintomática y la
linfadenopatía generalizada persistente.

A

Categoría A

351
Q

Clasificación de la infección por VIH: Incluye patologías no graves, se manifiestan en el principio de la fase avanzada, cuando el deterioro inmunológico no es tan grave.

A

Categoría B
*Patologías de categoría B: angiomatosis bacilar, candidiasis oral, candidiasis vulvovaginal de repetición,
displasia de cérvix de alto grado o carcinoma in situ, fiebre/diarrea de 1 mes, leucoplasia oral vellosa, herpes zóster de repetición o con afectación de >1 dermatoma, trombocitopenia, infección por Listeria y EPI.

352
Q

Clasificación de la infección por VIH: incluye las enfermedades oportunistas típicas de las fases más avanzadas de la enfermedad

A

Categoría C
*Patologías de categoría C: candidiasis traqueal
/bronquial/pulmonar/esofágica, carcinoma cervical
invasivo, coccidioidomicosis extrapulmonar, criptococosis extrapulmonar, criptosporidiosis intestinal
crónica, infección por CMV en órganos atípicos, retinitis por CMV, encefalopatía por VIH, herpes simple
con úlcera >1 mes, histoplasmosis diseminada, isosporiasis crónica, sarcoma de Kaposi, linfoma no
Hodgkin, infección por MAC, TBP o extrapulmonar, neumonía por P. jiroveci, neumonía recurrente,
Leucoencefalopatía multifocal progresiva, bacteriemia recurrente por salmonella, toxoplasmosis cerebral y síndrome de emaciación por VIH (SIDA clínico).

353
Q

Clasificación inmunológica de VIH: Categoría 1:

A

paciente con > 500 linfocitos TCD4

354
Q

Clasificación inmunológica de VIH: Categoría 2:

A

paciente con 200-499 linfocitos TCD4.

355
Q

Clasificación inmunológica de VIH: Categoría 3:

A

paciente con 200-499 linfocitos TCD4.

356
Q

El tratamiento de las úlceras por Candidiasis en VIH es

A

nistatina tópica. Las formas

graves requieren de fluconazol, equinocandinas o anfotericina B.

357
Q

es la causa más frecuente de meningitis en pacientes con SIDA

A

Cryptococcus neoformans:

358
Q

es un microorganismo ubicuo, pero característicamente sólo produce patología en pacientes con VIH y menos de 200 linfocitos TCD4.

A

Pneumocystis jiroveci

359
Q

es la causa más frecuente de convulsiones tras la encefalopatía por VIH y constituye la infección secundaria del SNC más habitual en los pacientes con SIDA

A

Toxoplasma gondii

360
Q

Toxoplasma gondii en px con VIH, La imagen característica en la TAC

A

una lesión redondeada con efecto de masa (Edema y compresión) que capta contraste en anillo.
Actualmente, la amplificación del ADN en LCR mediante PCR suele ser útil.

361
Q

Toxoplasma gondii en px con VIH: El tratamiento de primera elección es

A

sulfadiazina + pirimetamina por 6-8 semanas.

362
Q

En paciente con VIH produce cuadro de angiomatosis bacilar. Se diagnostica por biopsia (tinción de Warthin-Starry) o por hemocultivo. Se trata con eritromicina.

A

Bartonella henselae

363
Q

Causa un cuadro denominado Leucoencefalopatía
multifocal progresiva. Se presenta con diversos cuadros de afectación neurológica y con una imagen característica en RMN. La amplificación del genoma viral mediante PCR puede ser útil para el diagnóstico. El único tratamiento eficaz es la mejora del estado inmunológico del paciente mediante TARV.

A

Virus JC: perteneciente al género de Polyomavirus.

364
Q

Es el principal causante de hepatopatía crónica en pacientes con VIH

A

Hepatitis C

365
Q

Neoplasias asociadas a la infección por VIH

A

a. Neoplasias de órgano sólido: los carcinomas de cérvix y ano son especialmente frecuentes en pacientes con VIH y se relacionan con VPH. También aumenta la incidencia de melanoma cutáneo.
b. Linfomas: suelen ser de alto grado e inmunofenotipo B, como el linfoma inmunoblástico, el linfoma de
Burkitt o el linfoma cerebral primario (todos ellos categoría C).
c. Sarcoma de Kaposi: neoplasia con alta incidencia en pacientes con VIH. El VHH8 parece estar implicado
en su patogenia. Son lesiones de proliferación vascular típicamente cutáneas y mucosas, si bien pueden
afectar cualquier órgano. Se manifiestan como placas o nódulos de color violáceo, que en ocasiones
obligan a realizar diagnóstico diferencial con angiomatosis bacilar. La localización visceral más
frecuente es la intestinal y la de peor pronóstico la pulmonar. Según su localización y extensión el
tratamiento será más o menos agresivo, desde resección local, hasta radioterapia o quimioterapia
sistémica.

366
Q

Las combinaciones que actualmente se consideran de elección para tratar VIH son:

A
  • Dos ITRAN y un ITRNN.
  • Dos ITRAN y un inhibidor de la proteasa.
  • Dos ITRAN y un inhibidor de la integrasa.
    Los dos ITRAN que se consideran de elección son: emtricitabina + Tenofovir, Lamivudina + abacavir.
    El ITRNN de elección es el efavirenz.
    El inhibidor de la proteasa es el darunavir potenciado con ritonavir, ataznavir potenciado con ritonavir o bien Lopinavir potenciado con ritonavir.
    El atripla contiene emtricitabina, Tenofovir y efavirenz en una sola dosis diaria mediante preparado
    comercial.
367
Q

Tiña versicolor: está producida por

A

Malassezia furfur, un hongo lipofílico

368
Q

Tiña versicolor: CLÍNICA

A

Se localiza en tronco y cara en forma de zonas decoloradas en personas de piel oscura y zonas oscuras en personas de piel clara

369
Q

Tiña versicolor: TTO

A

vía tópica (ketoconazol, terbinafina, clotrimazol) o el tratamiento sistémico VO que se usa en formas más graves o de pobre respuesta al tratamiento tópico (itraconazol, fluconazol, ketoconazol).

370
Q

Dermatomicosis: son infecciones cutáneas que afectan a los tejidos queratinizados, incluyendo el pelo, piel
y uñas. Son conocidas como tiñas. Los agentes etiológicos pertenecen a los géneros

A

Trichophyton,

Microsporum y Epidermophyton

371
Q

Esporotricosis: es causada por el hongo dimórfico

A

Sporothrix schenkii. Tras un traumatismo, típicamente

pinchazo de un rosal

372
Q

Esporotricosis: CLINICA

A

se produce una úlcera que no cura y secundariamente se afectan los vasos linfáticos y ganglios linfáticos del territorio de drenaje

373
Q

Esporotricosis: El método de diagnóstico preferible es

A

el cultivo de pus, liquido articular, biopsia cutánea. Crece en agar Sabouraud.

374
Q

Esporotricosis: El tratamiento se realiza con

A

yoduro potásico o itraconazol

375
Q

La histoplasmosis es propia de zonas endémicas y es típica la adquisición tras

A

inhalación de esporas en cuevas contaminadas con excrementos de murciélagos. La mayoría cursa de forma asintomática, a veces puede formar masas pulmonares que pueden presentar calcificación.

376
Q

La coccidioidomicosis es una micosis profunda causada por un hongo dimórfico

A

Coccidioides immitis y posadasii
Se adquiere por inhalación de artroconidias y su espectro clínico es amplio. La infección es
usualmente benigna pero en pacientes con alteración de la inmunidad puede ser severa y fatal

377
Q

La coccidioidomicosis; diagnóstico definitivo se establece mediante la identificación del hongo mediante

A

cultivo (agar Sabouraud), las tinciones
que lo permiten ver en preparaciones son con hidróxido de potasio (más útil) y Grocot-Gomori. El método
más utilizado es la serología IgM e IgG contra antígenos capsulares de Coccidioides

378
Q

La coccidioidomicosis; TTO

A

nfotericina B para las formas severas. Los azoles son ideales para tratamientos crónicos.

379
Q

Mucormicosis: reúne a todas las infecciones causadas por hongos de la clase

A

Zygomycetes

380
Q

Mucormicosis: TTO

A

se fundamenta en tres pilares: antifúngico (anfotericina B y después posaconazol como mantenimiento), reversión del factor de riesgo y tratamiento quirúrgico.

381
Q

es la enfermedad parasitaria más importante en el ser humano y causa de 1-3 millones de muertes anuales

A

Paludismo

382
Q

Paludismo; El agente causal es transmitido por

A

la picadura de la hembra del mosquito Anopheles

383
Q

Paludismo; lásicamente se incluyen cuatro especies dentro del género Plasmodium que son

A

vivax, ovale, malariae y falciparum. La quinta especie descubierta es knowlesi capaz de causar enfermedad en el humano.

384
Q

Paludismo; Cuadro clínico

A

las infecciones en la edad adulta pueden ser asintomáticas.

  • Lo más frecuente es que cursan con cuadro prodrómico inespecífico (fiebre, cefalea, mialgias, artralgias, diarrea) que vayan seguidos de accesos palúdicos clásicos: fiebre, escalofríos y tiritonas a intervalos regulares.
  • No obstante, en la práctica clínica lo habitual es que la fiebre tenga un carácter más bien irregular.
  • A largo plazo pueden desarrollar anemia y esplenomegalia.
385
Q

Paludismo; Complicaciones crónicas:

A
  • Esplenomegalia palúdica hiperreactiva: producida por una reacción inmunitaria anormal, que se acompaña de hipergammaglobulinemia.
  • Nefropatía palúdica: asociada a P. malariae con síndrome nefrótico por depósito de inmunocomplejos
    con formación de glomerulonefritis focal y segmentaria.
386
Q

Paludismo; Complicaciones del paludismo grave:

A

Plasmodium falciparum, además de la destrucción de eritrocitos, provoca la adhesión de los mismos al endotelio vascular, por lo que tiene un curso más grave con trastornos circulatorios, sobre todo en cerebro y corazón.
- Paludismo cerebral: encefalopatía por trastorno circulatorio sanguíneo. Cursa con alteración del estado
de alerta o aparición de focalidad neurológica.
- Hipoglucemia: causada por el consumo de glucosa por parte del hospedero y el parásito, y fallo en la
gluconeogénesis hepática.
- Insuficiencia renal
- Otros: edema pulmonar no cardiogénico (alta mortalidad), trombocitopenia, CID, sepsis o acidosis
láctica.

387
Q

Paludismo; Diagnóstico:

A

se realiza mediante la visualización de las formas asexuales del parásito en una muestra de sangre periférica teñida con Giemsa (frotis de gota gruesa). También es útil la detección de antígeno palúdico en sangre por inmunocromatografía.
EL grado de parasitemia (número de eritrocitos parasitados por cada 1,000 células) tiene relación con el pronóstico.

388
Q

Paludismo; Tratamiento

A
  • P. falciparum sensible a cloroquina y P. vivax, ovale y malariae: cloroquina.
  • P. falciparum resistente a cloroquina: quinina + doxiciclina (en niños y embarazadas cambiar por
    clindamicina). En caso de la forma grave, se debe administrar por vía parenteral.
  • Se recomienda exanguinotransfusión cuando el grado de parasitemia es superior a 10% y el paciente
    presenta alteraciones neurológicas, edema pulmonar o fracaso renal.
  • En P. vivax y ovale, para los hipnozoítos, se utiliza primaquina asociada al tratamiento convencional
389
Q

Paludismo; Quimioprofilaxis

A

debe realizarse antes del viaje y continuarse después del regreso a una zona endémica.
En las formas sensibles a la cloroquina se utiliza este. En formas resistentes se utiliza Atrovacuona +
proguanilo.

390
Q

la leishmaniasis visceral o kala-azar está producida por especies del complejo

A

Leishmania donovani. También existe una forma cutánea (en México predominan las úlceras únicas en
áreas corporales expuestas).

391
Q

Leishmaniasis visceral: Clínica

A
  • la enfermedad visceral puede afectar a sujetos inmunocompetentes e inmunodeprimidos.
  • Son características la fiebre de predomino nocturno, esplenomegalia, pancitopenia, hipergammaglobulinemia e inmunocomplejos circulantes. También pueden existir adenopatías.
  • En las etapas más avanzadas hay edema e hiperpigmentación (fiebre negra).
392
Q

Leishmaniasis visceral: Diagnóstico

A

se utiliza la aspiración y biopsia de médula ósea para visualización de los amastigotes de Leishmania en el interior de los macrófagos, cultivo en medio NNN y serología. La prueba cutánea con reacción de Montenegro suele ser negativa en las formas viscerales.

393
Q

Leishmaniasis visceral: Tratamiento

A

anfotericina B liposomal.

394
Q

parásito de distribución mundial que se transmite por agua contaminada o de persona a persona por vía fecal oral. Es una de las etiologías de la diarrea del viajero.

A

Giardia lamblia (giardiasis

395
Q

giardiasis se Anida en

A

el duodeno y en intestino proximal y suele ser

asintomática.

396
Q

giardiasis Cuando se manifiesta incluye cuadros de

A

diarrea crónica con malabsorción, pérdida de peso,

flatulencia, náusea y diarrea intermitente.

397
Q

giardiasis Se diagnostica por

A

por la demostración del parásito en las heces o la

detección del antígeno.

398
Q

giardiasis se trata con

A

metronidazol o tinidazol

399
Q

El diagnostico de la amebiasis intestinal se realiza mediante

A

el examen directo de las heces, mientras que el absceso debe diagnosticarse por serología.

400
Q

El tratamiento de la infección intestinal como de la hepática por amebiasis debe incluir a

A

amebicida tisular (metronidazol, tinidazol o cloroquina) seguido de amebicida luminal para erradicar estado de portador asintomático (paramomicina, yodoquinol o diloxanida).

401
Q

Naegleria atraviesa la mucosa olfatoria y llega al SNC produciendo

A

meningoencefalitis aguda fulminante.

Se trata con anfotericina B liposomal

402
Q

Acanthamoeba y Balamuthia penetran por inhalación o la piel y provocan

A

meningoencefalitis granulomatosa clínica

403
Q

Enfermedad de Chagas: causada por Trypanosoma cruzi. Transmitida por

A

as heces de la chinche

besucona (Triatominae)

404
Q

Enfermedad de Chagas: clinica

A

La enfermedad aguda cursa con lesión inflamatoria en el área de entrada, acompañada de adenopatía regional. La fase crónica cursa con miocardiopatía dilatada (causa más frecuente de miocarditis infecciosa a nivel mundial) y los llamados mega síndromes (colon y esófago).

405
Q

Enfermedad de Chagas: Se diagnostica mediante y se

trata con

A

serología y se trata con bendzidazol o nifurtimox.

406
Q

Enfermedad del sueño: transmitida por

A

a mosca tsé-tsé (tripanosomiasis africana). Hay dos subespecies:
gambiense y rhodesiense.

407
Q

Enfermedad del sueño: clínica

A

Tiene una fase inicial con adenopatías, esplenomegalia y otra frase tardía con encefalitis.

408
Q

Enfermedad del sueño: Se diagnostica mediante

A

demostración del parásito en sangre, tejidos o LCR

(Giemsa) así como por serología

409
Q

Enfermedad del sueño: Se trata con

A

suramina, pentamidina o eflornitina.

410
Q

Babesiosis: agente etiológico.

A

Babesia microti
Se trata con Atrovacuona con
azitromicina.

411
Q

Teniasis: causadas por

A

T. solium (cerdo) y T. saginata (pescado). La ingesta de huevos ocasiona una infestación por la larva, en vez del parásito adulto, que produce la enfermedad denominada cisticercosis.
La cisticercosis afecta músculo y SNC y cursa con lesiones quísticas que evolucionan a calcificaciones y
pueden provocar crisis comiciales

412
Q

Teniasis: Se tratan con

A

praziquantel y albendazol.

413
Q

Oxiuriasis: también llamada enterobiasis. Es una infección causada por

A

Enterobius vermicularis. Se transmite por vía fecal oral y ocasiona prurito anal y perineal de predominio vespertino

414
Q

Oxiuriasis El diagnóstico se

establece mediante

A

la visualización de los huevecillos de los parásitos en una cinta adhesiva trasparente aplicada a los márgenes del ano (test de Graham).

415
Q

Oxiuriasis Se trata con

A

mebendazol, albendazol o Pamoato de pirantel.

416
Q

Estrongiloidiasis: producida por

A

Strongyloides stercoralis

417
Q

Strongyloides stercoralis ocasiona

A

infección pulmonar con infiltrados,
eosinofilia y diarreas. En inmunodeprimidos se produce forma severa invadiendo SNC, con hemorragias,
sepsis, meningitis o peritonitis

418
Q

Strongyloides stercoralis Se diagnostica por

A

examen de las heces

419
Q

Strongyloides stercoralis Se trata con

A

ivermectina

420
Q

Triquinosis: producida por

A

Trichinella spiralis, tras la ingestión de carne de cerdo poco cocinada

421
Q

Triquinosis Se manifiesta con

A

fiebre, mialgias, edema orbitario, hemorragias conjuntivales y ocasionalmente miocarditis
con eosinofilia

422
Q

Triquinosis Se diagnostica por

A

serología o biopsia muscular

423
Q

Triquinosis, tratamiento

A

En forma intestinal se usa mebendazol y

en la forma de miositis o miocarditis esteroides.

424
Q

la uncinariasis o anquilostomiasis es una parasitosis producida por

A

Ancylostoma duodenale o Necator americanus. Produce anemia ferropénica e hipoproteinemia en pacientes desnutridos. Se trata con albendazol.

425
Q

Hidatidosis: la enfermedad es causada por la forma larvaria de

A

Echinococcus granulosus. Los perros son los hospederos definitivos, almacenan los gusanos adultos en su intestino. Los huevos salen con las heces y pasan al hospedero intermediario. La mayoría de las infecciones son en la niñez producidas por la ingestión de material contaminado por las heces de perro. En el 70% de los casos produce afectación hepática por lo general en lóbulo derecho.

426
Q

Hidatidosis Cuadro clínico:

A

el parásito tiene crecimiento a nivel visceral y no suele dar síntomas, cuando los da, lo más común es dolor e hipersensibilidad abdominal. En ocasiones se palpa masa en hipocondrio derecho o hepatomegalia. La forma pulmonar suele ser hallazgo radiológico, pero puede provocar dolor torácico y vómica.

427
Q

Hidatidosis Complicaciones:

A
  • Rotura de la vía biliar: complicación más frecuente con cólico biliar, ictericia y prurito.
  • Sobreinfección.
  • Rotura a la cavidad peritoneal.
  • Perforación intratorácica.
428
Q

Hidatidosis Diagnóstico:

A

se utiliza analítica (eosinofilia periférica), ecografía/TAC (masa hepática) y prueba de aglutinación directa o el test de Cansoni.

429
Q

Hidatidosis Tratamiento:

A

la PAAF con ecografía y aspiración con inyección de alcohol o salino hipertónico es eficaz y seguro, se recomienda realizar de forma simultánea con tratamiento con albendazol.

430
Q

Fasciola hepática: se adquiere a partir de la ingestión de

A

erros o agua contaminada. Produce fiebre y clínica
digestiva y hepática con marcada eosinofilia. Se diagnostica por serología o detección del parásito en heces. Se trata con triclalbendazol.

431
Q

microfilariasis linfática producida por

A

Wuchereira bancrofti y Brugia malayi: microfilariasis linfática. Clínicamente presentan elefantiasis. Se
trata con ivermectina.

432
Q

Onchocerca volvulus: cuadro cutáneo por

A

microfilarias con prurito, despigmentación cutánea, ceguera y coriorretinitis. Se trata con ivermectina

433
Q

Loa loa: filariasis cutánea, transmitida por

A

picadura de tábano. Ocasiona edema migratorio acompañado de eosinofilia y conjuntivitis. Se trata con ivermectina.

434
Q

Esquistosomiasis: la infección por

A

Schistosoma mansoni es endémica en áfrica

435
Q

La infección por Schistosoma haematobium produce

A

una parasitación de la vejiga urinaria, siendo responsable en ocasiones de aparición de carcinoma vesical de células escamosas. Se diagnostica por estudio de heces u orina. Se trata con praziquantel.

436
Q

Anisakiasis: producida por la ingestión de

A

escado rudo o poco cocido contaminado por Anisakis simplex que parasita la pared gástrica y ocasiona dolor abdominal, náuseas y vómitos al cabo de 12-48 horas. Puede ocasionar obstrucción intestinal. Se diagnostica y trata mediante endoscopia.